Download as pdf or txt
Download as pdf or txt
You are on page 1of 77

Three Dimensional

UNIT 5 THREE DIMENSIONAL GEOMETRY Geometry

Structure
5.1 Introduction
Objectives

5.2 Coordinates of a Point in Space


5.3 The Plane
5.4 Equations of a Straight Line (Definitions and Properties)
5.5 Block and Cube
5.6 Sphere, Cone and Cylinders
5.7 Areas and Volumes
5.8 Numerical Integration and Simpson’s Rule
5.9 Summary
5.10 Answers to SAQs

5.1 INTRODUCTION
We know that in the study of co-ordinate geometry of two dimensions, the position of a
point is determined by an ordered pair of numbers (x, y) referred to two intersecting lines
mutually perpendicular to each other and described as axes of co-ordinates. The physical
activity associated to a plane such as flow of fluid along a plane or flow of heat along a
bar can be studied with the help of the two dimensional geometry. The straight line and
other conics confined in a two dimensional configurations require for their study the
location of points on them which can be described by an ordered pair (x, y) in Cartesian
co-ordinates or by (r, θ) defined as polar co-ordinates of the points. In the description of
solid objects, however, different points on them do not lie in the same plane and hence
the system involving two dimensional geometry becomes insufficient to describe the
position of various points on the solid. In the study of solid objects, therefore, it becomes
imperative to extend the system and enlarge the domain to take care of outer boundaries
of solids and other surfaces. Hence this study of surfaces of solids becomes important
from physical point of view and this constitutes the main subject matter of geometry of
three dimensions.
Objectives
After studying this unit, you should be able to
• define the co-ordinates of a point in space, the direction cosines of a line and
find the angle between two lines,
• find the equations of a plane in various forms and also the perpendicular
distance of a point from a plane,
• state the equation of a sphere in different forms and study the section of a
sphere by a plane,
• write the equation of a cone, a right circular cone, a cylinder and a right
circular cylinder,
• discuss analytically the equations involving polynomials with one variable,
with two variables and with three variables,
• find out areas and volumes, volumes as double integrals and volume as
triple integrals,

51
Mathematics-I • define three systems of co-ordinates, the Cartesian system (x, y, z), the
cylindrical co-ordinates (ρ, φ, z) and the spherical polar co-ordinates
(r, θ, φ), and
1 3
• derive trapezoidal rule, Simpson’s rule, Simpson’s rule and use these
3 8
methods of numerical integration in the computation of areas, volumes and
the centre of gravity of the bodies.

5.2 COORDINATES OF A POINT IN SPACE


As we are representing a three dimensional configuration in a two dimensional plane, we
take y-axis normal to the XZ-plane. Let XOX′, YOY′, ZOZ′ be three mutually
perpendicular lines through the origin. The lines XOX′, YOY′, ZOZ′ are called the axes of
co-ordinates or more briefly the co-ordinate axes. The three axes taken in pairs define
three planes usually characterized as xy, yz, and zx-planes. Together these planes are
called co-ordinate planes. These three planes divide the entire space into
8 (2 × 2 × 2) compartments known as octants. The plane OXYZ in which all the three
space co-ordinates are positive is called the first quadrant. The procedure of locating the
position of a point in space is as under.
Z
z x-plane
z-axis Y′

y z-plane P

x-axis M
X′ X
O
y-axis
N
x y-plane
Y′
Z′

Figure 5.1
Let P be any point in space. From P draw PN perpendicular to xy-plane. Through N draw
NM parallel to y-axis to meet the x-axis at M, then OM = x, MN = y and NP = z. The
co-ordinates of the points M, N and P referred to O as the origin are denoted by (x, 0, 0),
(x, y, 0), (x, y, z) respectively.
Important to Remember
(a) The Z-coordinate of every point N in the xy-plane is always zero. Therefore
the equation of xy-plane is z = 0. Similarly x = 0 and y = 0 are the equations
of yz and zx planes respectively.
(b) Every point M on the x-axis has both its y and z co-ordinates as zero. Hence
the equation of x-axis is given by y = 0 and z = 0. In other words x-axis is
the intersection of the two planes xz = 0 and xy = 0. Similarly the equations
of y-axis and z-axis are given by x = 0, z = 0 and x = 0, y = 0 respectively.
5.2.1 Projection on a Line
The projection of a line segment AB on another line CD is the line segment A′B′ where A′
and B′ are the feet of the perpendicular from A, B on the line CD. In other words A′B′=
AB × cosine of the angle between AB and CD.
5.2.2 Direction Cosines of a Line
If α, β, γ are the angles which a line OP makes with the positive directions of the x, y and
z-axes respectively, then the quantities cos α, cos β, cos γ which provide the information
about certain direction in space are called the direction cosines of the line OP.
52
Let P (x, y, z) be any point in space. From P drop perpendiculars on OX, OY and OZ and Three Dimensional
let them meet the axes in N, S and T respectively. Geometry

Then by the theorem of three perpendiculars, PN is perpendiculars on OX and


ON
∴ cos α =
OP
Similarly by dropping perpendiculars from the P on OY and OZ, we get
OS
cos β =
OP
OT
and cos γ =
OP
Z

P (x, y, z)
T

γ
z
α
O x
x N (x, 0, 0)
β y

M (x, y, 0)

Y
Figure 5.2

Note
The direction cosines of the line are invariably denoted by the letters l, m, n
respectively, i.e. cos α = l , cos β = m and cos γ = n . The direction cosines are
sometimes abbreviated as (d . cs).
Remarks
The direction cosines of x, y and z axes are
(1, 0, 0), (0, 1, 0), (0, 0, 1) ⎡cos 0o , cos 90o , cos 90o ⎤ etc.
⎣ ⎦
A Useful Result
If l, m, n are the d . cs of the line OP and if the length OP = r, then the coordinates
of the point P are (l r, m r, n r).
Let (x, y, z) be the coordinates of the point P and let M be the point where the
perpendicular from P meets the x-axis. Then OM = x = OP cos ∠ POM =
r cos α = r l. From similar considerations y = m r, z = n r.
∴ The co-ordinates of P are (l r, m r, n r).
Corollary 1
If r = 1, then the co-ordinates of P are (l, m, n).
(A) Relation between the Direction Cosines of a Line
If l, m, n are the direction cosines of a line, then
l 2 + m 2 + n2 = 1 .

53
T
P (x, y, z)

S
Mathematics-I

Figure 5.3

Let ST be the line whose direction cosines are l, m, n. From O draw a line Op
parallel to ST. Since direction cosines of parallel lines are equal, the (d . cs) of OP
are also l, m, n. Let the co-ordinates of P be (x, y, z). Then OP 2 = x 2 + y 2 + z 2 .
Now OM = x = Projection of OP on OX = OP cos ∠ POX = OP . l.
Similarly y = OP . m and z = OP . n.
Substituting for x, y, z in the relation above, we get
OP 2 = x 2 + y 2 + z 2 = OP 2 l 2 + OP 2 m 2 + OP 2 n 2

This gives l 2 + m 2 + n 2 = 1 , i.e. the sum of the squares of the direction cosines of
a line is unity (Analogy with plane geometry cos 2 α + sin 2 α = 1 ).

5.2.3 Direction Ratios


The direction ratios of a line are numbers proportional to the direction cosines of the line.
If l, m, n are the direction cosines of a line, then kl, km, kn are the direction ratios of the
line, k being other than zero and unity.
If a, b, c are the direction ratios of a line such that a = k l; b = k m; c = k n then squaring
and adding, we get
a 2 + b2 + c2 = k 2 l 2 + k 2 m2 + k 2 n2 = k 2

∴ k = a2 + b2 + c2
Substituting for k, we get
a b c
l= ,m= ,n=
k k k
a b c
or l= ,m= ,n= . . . (1)
∑ a2 ∑ a2 ∑ a2
Hence if the direction ratios of a line are given as a, b, c to obtain the direction cosines of
the line, divide its direction ratios by the square root of the sum of the squares of the
direction ratios.
Direction Cosines of a Line Joining Two Points
To find the direction cosines of a line joining two points
P ( x1 , y1 , z1 ), Q ( x2 , y2 , z2 )
Let l, m, n be the direction cosines of the line PQ. Now (x2 − x1) which is the
intercept on the x-axis is the projection of PQ on x-axis.
Also the projection of PQ on x-axis is PQ . l.
∴ x2 − x1 = PQ . l

54
x2 − x1 Three Dimensional
∴ l= Geometry
PQ
y2 − y1 z − z1
Similarly m = and n = 2 .
PQ PQ

But PQ 2 = ( x2 − x1 ) 2 + ( y2 − y1 ) 2 + ( z2 − z1 ) 2 = Square of the length


PQ = a fixed length.
x2 − x1 y2 − y1 z2 − z1
∴ = = = PQ
l m n
Hence the direction cosines of the line joining two points
P ( x1 , y1 , z1 ), Q ( x2 , y2 , z2 )
are proportional to
x2 − x1 ; y2 − y1 ; z2 − z1 . . . (2)
i.e. the difference of x-coordinates, difference of y-coordinates, difference of
z-coordinates.
Angle between Two Lines Whose Direction Cosines are Given as
( l1 , m1 , n1 ) and ( l 2 , m2 , n2 )
Z
D
B
C
N A

M
θ
X
O

Y
Figure 5.4

Let AB and CD be two lines and let θ be the angle between them. Through the
origin O draw two lines OM and ON parallel to the given lines so that the direction
cosines of OM and ON are the same as these of the given lines.
i.e. (l1 , m1 , n1 ) and (l2 , m2 , n2 )

Since θ is the angle between the given lines and OM and ON are parallel to them,
the angle between OM and ON will also be θ.
Now without any loss of generality, we assume both OM and ON to be unit length
so that OM = ON = 1.
The co-ordinates of M and N are (l1 , m1 , n1 ) and (l2 , m2 , n2 ) [refer Corollary 1].

In the triangle OMN, we have

MN 2 = OM 2 + ON 2 − 2OM × ON cos θ

i.e. [(l2 − l1 ) 2 + ( m2 − m1 ) 2 + (n2 − n1 ) 2 ] = 1 + 1 − 2 cos θ = 2 − 2 cos θ

Simplifying the left side by using the property that the sum of the squares of the
direction cosines is 1, we get
2 − 2 (l1 l2 + m1 m2 + n1 n2 ) = 2 − 2 cos θ

i.e. cos θ = l1 l2 + m1 m2 + n1 n2

Corollary 2
55
Mathematics-I
sin 2 θ = 1 − cos 2 θ = 1 − (l1 l2 + m1 m2 + n1 n2 ) 2

= (l12 + m12 + n12 ) (l22 + m22 + n22 ) − (l1 l2 + m1 m2 + n1 n2 ) 2


2 2 2
l m1 m n1 n l1
= 1 + 1 + 1
l2 m2 m2 n2 n2 l2

∴ sin θ = ± ∑ (l1 m2 − l2 m1 ) 2
Corollary 3
If the direction cosines of two lines are proportional to (a1, b1, c1) and ( a2 , b2 , c2 ) ,
then the actual direction cosines of the two lines are
a1 b1 c1
± ,± ,±
∑ a12 ∑ a12 ∑ a12
a2 b2 c2
and ± ,± ,±
∑ a22 ∑ a22 ∑ a22
a1 a2 + b1 b2 + c1 c2
∴ cos θ =
∑ a12 ∑ a22

∑ (a1 b2 − a2 b1 )2
and sin θ =
∑ a12 ∑ a22
Corollary 4 : Condition for Perpendicularity
When two lines are perpendicular, angle between them is 90o,
cos 90o = 0 ∴ a1 a2 + b1 b2 + c1 c2 = 0
Corollary 5 : Condition for Parallelism
Let the direction ratios of two lines be (a1 , b1 , c1 ) and ( a2 , b2 , c2 ) . If the two lines
are parallel, their direction cosines are equal. This gives
a1 a2 b1 b2 c1 c2
= ; = ; =
∑ a12 ∑ a22 ∑ a12 ∑ a22 ∑ a12 ∑ a22
a1 b1 c1
Hence = =
a2 b2 c2
Projection of the Join of Two Points on a Line Whose Actual Direction Cosines
are Given
To find the projection of the join of two points P ( x1 , y1 , z1 ) and Q ( x2 , y2 , z2 )
on other line whose actual direction cosines are given.
Let us suppose that θ is the angle which the line PQ makes with the line whose
direction cosines are (l, m, n). The direction ratios of PQ are x2 − x1 , y2 − y1 ,
z2 − z1 (refer Corollary 2) and consequently direction cosines of PQ are
x2 − x1 y2 − y1 z2 − z1
, , .
PQ PQ PQ
l ( x2 − x1 ) + m ( y2 − y1 ) + n ( z2 − z1 )
∴ cos θ =
PQ

But the projection of PQ on the given line is PQ cos θ.


56
= l ( x2 − x1 ) + m ( y2 − y1 ) + n ( z2 − z1 ) Three Dimensional
Geometry
Corollary 6
If P is the point ( x1 , y1 , z1 ) , then the projection of OP on a line whose direction
cosines are (l, m, n) is l x1 + m y1 + n z1 .
Example 5.1
A line makes angles of 45o and 60o with the positive axes of x and y respectively,
what angle does it make with the positive axis of z?
Solution
Let l, m, n be the direction cosines of the line. This line makes angles of 45o and
60o with the positive directions of x and y axes respectively. Suppose the line
makes an angle θ with the z-axis.
We have know
1 1
l = cos 45o = ; m = cos 60o = and cos θ = n
2 2
The direction cosines of x, y and z-axes are (1, 0, 0), (0, 1, 0), (0, 0, 1)
[Note 5.2.2].
Using cosine formula for the angle, we get
1
cos 45o = = l ×1+ m × 0 + n × 0
2
1
∴ l=
2
1
Similarly m=
2
and n = cos θ .

But l 2 + m2 + n2 = 1
Substituting for l, m, we get
1 1
+ + n2 = 1
2 4
1
or n2 =
4
1
n=± = cos θ
2
∴ Q = 60o or 120o
Example 5.2
Find the direction cosines of a line which makes equal angles with the axes. How
many such lines are there?
Solution
Let α be the angle made by the line with the axes. Then
cos α = l × 1 + m × 0 + n × 0 = l × 0 + m × 1 + n × 0 = l × 0 + m × 0 + n × 1

l 2 + m2 + n2 1
i.e. l=m=n= =±
3 3

57
Mathematics-I 1 1 1
∴ If α is the acute angle, the direction cosines of the line are , , ;
3 3 3
Now if l, m, n are the d . cs of PQ, − l, − m, − n are the d.cs of
QP [cos (180o − α ), cos (180o − β), cos (180o − γ )]
∴ The direction cosines of the line are
1 1 1
± ,± ,± ;
3 3 3
This gives in fact 4 such distinct lines.
Example 5.3
The projections of a line on the axes are 3, 4, 12. What is the length of the line?
Solution
Let the direction cosines of the line be l, m, n. The projection of this line on the
x-axis (the d . cs of x-axis are 1, 0, 0 ) is given as 3.
∴ 3 = 1 × l ′ + 0 × m′ + 0 × n′ = l ′
Similarly 4 = m′ and 12 = n′ [Corollary 6] where l ′, m′, n′ are the direction ratios
of the line.
Now the length of projection = l × 1 + m × 1 + n × 1
l′ l′ m′ n′
Where l= = ,m= ,n=
32 + 42 + 122 13 13 13

or 13 = length of projection = 1 × l ′ + 1 × m′ + 1 × n′
Example 5.4
Lines OP and OQ are drawn from O with direction cosines proportional to
(1, − 1, − 1) and (2, − 1, 1) . Find the direction cosines of the normal to the plane
POQ.
Solution
Let the direction cosines of the normal to the plane POQ be l, m, n. Since the lines
OP and OQ lie in the plane, the normal to the plane will also be normal to the
lines. Using the condition of perpendicularity, we get
l−m−n=0
2l − m + n = 0
Solving for proportional values, we get

l m n l 2 + m2 + n2 1
= = = =±
−1−1 − 2 −1 −1+ 2 14 14
2 3 1
∴ l= ,m= ,n=−
14 14 14
Example 5.5
Light from a point source at A (7, 0, 0) strikes a small mirror at the origin O, the
normal to which has direction ratios 2 : 1 : 2. Find the actual direction cosines of
the reflected ray.
Solution
Let l, m, n be the direction cosines of the reflected ray so that l 2 + m 2 + n 2 = 1 .
The normal bisects the angle between the reflected ray and OA, the x-axis.
58
The direction cosines of x-axis are (1, 0, 0). The direction cosines of the line Three Dimensional
bisecting the angle between OA and the reflected ray are proportional to Geometry
l + 1, m + 0, n + 0 .
l +1 m n
Hence = = = λ (say).
2 1 2
This gives l = 2λ − 1, m = λ, n = 2λ .
Substituting for l, m, n in the above equation, we get
4
(2λ − 1)2 + λ 2 + 4λ 2 = 1 or λ = 0,
9
⎛ 1 4 8⎞
∴ The direction cosines l, m, n of the reflected ray are ⎜ − , , ⎟.
⎝ 9 9 9⎠
Some Useful Results (Without Proof)
(a) Distance between Two Points (Distance Formula)
If P and Q are two points in space distance R apart and if their coordinates
are ( x1 , y1 , z1 ) and ( x2 , y2 , z2 ) then

R 2 = ( x2 − x1 ) 2 + ( y2 − y1 )2 + ( z2 − z1 ) 2
If one of the points say Q coincides with the origin then
x2 = y2 = z2 = 0

and R 2 = x12 + y12 + z12


(b) Ratio Formula
The co-ordinates (x, y, z) of a point which divides the join of
P ( x1 , y1 , z1 ) and Q ( x2 , y2 , z2 ) in the ratio of m : n are
m x2 + n x1 m y2 + n y1 m z2 + n z1
x= ;y= ;z=
m+n m+n m+n
Corollary 7
The co-ordinates of the mid point of the points P ( x1 , y1 , z1 ) and Q ( x2 , y2 , z2 )
are
x1 + x2 y + y2 z + z2
x= ;y= 1 ;z= 1
2 2 2
Corollary 8
The co-ordinates of any point R ( x, y , z ) which divides the join of P ( x1 , y1 , z1 )
and Q ( x2 , y2 , z2 ) in the ratio of λ : 1 are
λ x2 + x1 λ y2 + y1 λ z2 + z1
x= ;y= ;z=
λ +1 λ +1 λ +1
SAQ 1
(a) If α, β, γ are the angles which a line makes with the axes, prove that
sin 2 α + sin 2 β + sin 2 γ = 2 .
(b) The direction cosines of a line are proportional to 2, 3, 6. A point P on a
parallel line through the origin is distant 14 units from the origin. Find its
co-ordinates.
(c) Find the ratio in which the line joining the points (2, 4, 16) and (3, 5, − 4) is
divided by the plane 2 x − 3 y + z + 6 = 0 . Determine also the co-ordinates
of the point of division.
59
Mathematics-I (d) A and B are two points having their co-ordinates (1, 2, − 1) and (2, 1, 1).
Find the projection of AB on the line CD whose direction ratios are
(1, 1, − 1).

5.3 THE PLANE


Definition
A plane is defined as a surface such that if any two points are taken on the surface,
the line joining them lies wholly on the surface.
5.3.1 General First Degree Equation
To prove that every equation of the first degree in x, y, z represents a plane.
The general equation of the first degree in x, y, z is
ax + by + cz + d = 0 . . . (1)
Let ( x1 , y1 , z1 ) , ( x2 , y2 , z2 ) be any two points lying on Eq. (1).
Then ax1 + by1 + cz1 + d = 0 . . . (2)
ax2 + by2 + cz2 + d = 0 . . . (3)
The Eq. (1) will represent a plane if every point on the line joining ( x1 , y1 , z1 ) and
( x2 , y2 , z2 ) lies on the locus of Eq. (1).
Now the co-ordinates of any point on the line joining these points ( x1 , y1 , z1 ) and
( x2 , y2 , z2 ) are
kx2 + x1 ky2 + y1 kz2 + z1
, ,
k +1 k +1 k +1
This point will lie on Eq. (1) if
⎛ kx + x1 ⎞ ⎛ ky2 + y1 ⎞ ⎛ kz2 + z1 ⎞
a⎜ 2 ⎟+b⎜ ⎟+c⎜ ⎟+d =0
⎝ k +1 ⎠ ⎝ k +1 ⎠ ⎝ k +1 ⎠
or if k ( ax2 + by2 + cz2 + d ) + ( ax1 + by1 + cz1 + d ) = 0 . . . (4)
Relation (4) is zero by virtue of (2) and (3) and for all values of k.
Hence (1) represents a plane. It is called the general form of the equation of the plane.
Corollary 9
The equations of co-ordinate planes XOY, YOZ and ZOX are
z = 0; x = 0; y = 0.
Important
The General Equation of the plane ax + by + cz + d = 0 in Analytical Solid
Geometry bears a close analogy with the equation of the straight line
ax + by + c = 0 in analytical plane geometry.
5.3.2 Various Forms of the Equations of the Plane
Here under we discuss the various forms of the equation of the plane.
One-Point Form
One-point form, i.e. the plane passing through one-point ( x1 , y1 , z1 ) .
60
Let the general equation of the plane be ax + by + cz + d = 0 . Since the plane Three Dimensional
Geometry
passes through ( x1 , y1 , z1 ) , we get
ax1 + by1 + cz1 + d = 0
Eliminating d from the above equations, we get
a ( x − x1 ) + b ( y − y 1 ) + c ( z − z1 ) = 0
This is the required equation of the plane in one-point form.
Equation Through Three Points
Let the co-ordinates of the three points be ( x1 , y1 , z1 ) , ( x2 , y2 , z2 ) , ( x3 , y3 , z3 )
and let the general equation of the plane be
ax + by + cz + d = 0
Since the plane passes through three points, we get
ax1 + by1 + cz1 + d = 0
ax2 + by2 + cz2 + d = 0
ax3 + by3 + cz3 + d = 0
Eliminating a, b, c, d from the above four equations, we get
x y z 1
x1 y1 z1 1
=0
x2 y2 z2 1
x3 y3 z3 1
which is the equation of the plane.
Example 5.6
Find the equation of the plane through three points
(0, 1, 1), (1, 1, 2), (− 1, 2, − 2).
Note : To find the equation of the plane by the above method, it is cumbersome to
expand a fourth order determinant. When numerical values of the
co-ordinates are given, we use a simple method as explained hereunder.
Solution
The equation of any plane through (0, 1, 1) [One-point form] is given as
a ( x − 0) + b ( y − 1) + c ( z − 1) = 0
If this plane passes through (1, 1, 2) and (− 1, 2, − 2), then
a + b (0) + c (1) = 0 ⇒ a + 0×b + c = 0
a (− 1) + b (1) + c (− 3) = 0 ⇒ − a + b − 3c = 0
Solving for a, b, c by across-multiplication, we get
a b c a b c
= = or = =
0 −1 −1+ 3 1+ 0 −1 2 1
Substituting for a, b, c for proportionate values in the equation of one-point form,
we get
− x + 2 ( y − 1) + ( z − 1) = 0
or x − 2y − z + 3 = 0
Normal Form of the Equation of the Plane
To find the equation of a plane in terms of p, the length of the perpendicular from
the origin on the plane, and l, m, n, the direction cosines of this perpendicular.
61
Mathematics-I Let ABC be the plane and OL the perpendicular from O on it so that OL = p and l,
m, n are the direction cosines of OL.
Z

P (x, y, z)

L
X
O A

Figure 5.5
Let P ( x, y , z ) be any point on the plane.
Now, OL = Projection of OP on OL (Ref. Section 5.1.3, Corollary 6)
= l ( x − 0) + m ( y − 0) + n ( z − 0)
= lx + my + nz
But OL = P
∴ p = lx + my + nz
which is the required equation of the plane. The equation is known as the normal
form of the equation of the plane.
Corollary 10
The equation of the plane will be in the normal form if the sum of the squares of
the coefficients of x, y, z is unity, i.e. l 2 + m 2 + n 2 = 1 as l, m, n are the actual
direction cosines of OL, the normal to the plane.
Note : p must always be positive.
Important : In the equation of a plane ax + by + cz + d = 0 the coefficients of
x, y, z, i.e. a, b, c are proportional to the direction cosines of the
normal to the plane.
Example 5.7
The direction cosines of the perpendicular from the origin to the plane are
proportional to 1, 3, 1 and the length of the perpendicular is 2 units. Find the
equation of the plane.
Solution
The direction cosines of the normal to the plane from the origin are
1 3 1
, , and the length is given as 2. The equation of the required plane
11 11 11
is, therefore, given as
x 3y z
+ + =2
± 11 ± 11 ± 11

or x + 3 y + z = 2 11
62
Corollary 11 Three Dimensional
Geometry
(Most Important Result). Rule to reduce the general equation
ax + by + cz + d = 0 of the plane to the normal form l x + m y + n z = p .

(a) Divide the general equation throughout by a 2 + b2 + c2

i.e. (coeff. of x) 2 + (coeff. of y ) 2 + (coeff. of z )2

(b) Transpose the constant term to the RHS and make it positive by changing
the sign throughout, if necessary. The transformed equation will be the one
in normal form.
Example 5.8
Find the direction cosines of the perpendicular drawn from the origin to the plane
2x + 3y − z + 1 = 0 .
Solution
We have seen that the coefficients of x, y, z in the equation of the plane are
proportional to the direction ratios of the normal to the plane. Therefore 2, 3, − 1
are the direction ratios of the normal to the plane 2 x + 3 y − z + 1 = 0 .
The actual direction of the normal are
2 3 −1
± ,± ,±
14 14 14
Writing the equation of the plane in the normal from, i.e. 2 x + 3 y − z = − 1 .
Since the RHS must be positive, the actual direction cosines of the normal are
−2 −3 1
, ,
14 14 14
The sign has been adjusted to make the right hand side positive.
Equation of the Plane in the Intercept Form
To find the equation of the plane which cuts off intercepts a, b, c on the axes of co-
ordinates.
Let ABC be the plane meeting the axes of the co-ordinates in A, B, C so that
OA = a; OB = b; OC = c .
Z
C

a
b X
O A

B
Y
Figure 5.6

Let the required equation of the plane be


αx + βy + γz + d = 0
because the plane meets the axes in A, B and C, the co-ordinates of these points
will be
(a, 0, 0), (0, b, 0), (0, 0, c )
Substituting these points in the equation, we get 63
Mathematics-I a α + d = 0; b β + d = 0; cγ + d = 0
From the above relations, we obtain
d d d
α=− , β=− , γ=−
a b c
Substituting these values of α, β, γ in the equation of the plane and dividing
throughout by d, we get
x y z
− − − +1= 0
a b c
x y z
or transposing + + =1
a b c
which is the required equation of the plane.
x y z
Note : Since in the equation + + = 1 , a, b, c are the intercepts of the plane
a b c
on the axes, this form of the equation of the plane is called the intercept
form.
General Equation of a Plane, Some Particular Cases
Planes parallel to the Co-ordinate Planes
If we draw a plane parallel to YOZ plane at a distance ‘a’ from it, then every
point in this plane will have its x-coordinates as ‘a’ and therefore equation
of the plane can be taken as x = a. Similarly plane parallel to ZOX plane at a
distance ‘b’ can be written as y = b and a plane parallel to XOY at a distance
‘c’ from it will have its equation z = c.
Planes Parallel to Co-ordinate Planes
Consider the general plane
ax + by + cz + d = 0 . . . (1)
If it is parallel to x-axis, normal to the plane (1) will be perpendicular to
x-axis. The direction ratios of x-axis are (1, 0, 0) and that of the plane (1) are
(a, b, c).
∴ a . 1 + b . 0 + c . 0 = 0 or a = 0
(condition of perpendicularity).
Putting a = 0 in (1), the plane parallel to x-axis will have its equation as
by + cz + d = 0
Important : Thus if x term is absent in the equation of a plane, then it is
parallel to x-axis.
Similarly, planes parallel to y and z-axes will have their equations as
ax + cz + d = 0
and ax + by + d = 0
Planes Perpendicular to Co-ordinate Planes
Any plane perpendicular to YOZ-plane is parallel to the x-axis and its
equation can be taken as
by + cz + d = 0

64
Similarly, we can take planes perpendicular to ZOX and XOY planes as Three Dimensional
ax + cz + d = 0 and ax + by + d = 0 respectively. Geometry

Example 5.9
Find the intercepts made on the co-ordinate axes by the plane 2 x + y − 2 z = 3 .
Find also the direction cosines of the normal.
Solution
The plane meets the x-axis where y and z co-ordinates are zero.
3
∴ 2 x = 3 or x =
2
3
Similarly y = 3 and z = − .
2
3 3
Hence the intercepts on the axes are , 3, − . The direction ratios of the normal
2 2
to the plane are 2, 1, − 2 .

∴ The actual direction cosines are


2 1 −2
, ,
3 3 3
Example 5.10
Find the equation of the plane which passes through the point (2, − 3, 1) and is
normal to the line joining the points (2, − 1, 5), (3, 4, − 1) .
Solution
The equation of the plane in one-point form is given by
a ( x − 2) + b ( y + 3) + c ( z − 1) = 0
The direction ratios of normal to this plane are a, b, c. The direction ratios of the
line joining the points (2, −1, 5) and (3, 4, − 1) are (1, 5, − 6) .
Since the plane is normal to the line, the normal to the plane is parallel to this line.
a b c
∴ = =
1 5 −6
Substituting for a, b, c for proportionate values, the equation of the plane is given
by
1 ( x − 2) + 5 ( y + 3) − 6 ( z − 1) = 0
or x + 5 y − 6 z + 19 = 0

5.3.3 Plane Through the Intersection of Two Planes


To find the equation of a plane passing through the line of intersection of the two planes
whose equations are
u = ax + by + cz + d = 0 . . . (1)
v = a1 x + b1 y + c1 z + d1 = 0 . . . (2)
Consider the equation
ax + by + cz + d + k ( a1 x + b1 y + c1 z + d1 ) = 0 . . . (3)
or u + kv = 0
65
Mathematics-I The Eq. (3) being of the first degree in x, y z represents a plane. It is also satisfied by the
co-ordinates of all those points which satisfy u = 0 and v = 0 simultaneously.
Hence the Eq. (3) defines a plane which goes through the common points of u = 0 and
v = 0, i.e. the line of intersection of these two planes.
Therefore, equation u + kv = 0 is the equation of a plane passing through the line of
u = 0 and v = 0.
Example 5.11
Find the equation of the plane which contains the line of intersection of the planes.
6x + 4 y − 5z = 2
and x − 2 y + 3z = 0
and is perpendicular to the plane 3 x − 2 y + z = 5 .
Solution
The equation of the plane containing the line of intersection of the given planes is
6 x + 4 y − 5 z − 2 + k ( x − 2 y + 3 z ) = 0 [u + k v = 0]
or (6 + k ) x + (4 − 2k ) y + (3k − 5) z − 2 = 0
The direction ratios of the normal to this plane are
(6 + k , 4 − 2k , 3k − 5)
Now this plane whose direction ratios are (6 + k , 4 − 2k , 3k − 5) will be
perpendicular to the plane 3 x − 2 y + z = 5 if the normals are perpendicular
∴ 3 (6 + k ) − 2 (4 − 2k ) + (3k − 5) = 0
1
or 10k = − 5 or k = −
2
Putting this value of k, the equation of the plane is given by
1
6 x + 4 y − 5z − 2 − ( x − 2 y + 3z) = 0
2
or 11x + 10 y − 13 z − 4 = 0
5.3.4 Angle between Two Planes
Definition
Let p and q be the two planes and LM their line of intersection. At any point P in
LM, draw two perpendiculars, PQ perpendicular to LM in the plane p, and PR
perpendicular to LM in the plane q. Then ∠ QPR is the angle which the plane p
makes with the plane q, more briefly the angle between the two planes p and q.

p
Q
A
L

P R

M q

Figure 5.7

66 Corollary 12
The angle between two planes is the angle between their normals (Proof is self Three Dimensional
explanatory from the figure.) Geometry

Let the equations of the two places be


a1 x + b1 y + c1 z + d1 = 0
a2 x + b2 y + c2 z + d 2 = 0
The angle between the two planes = angle between their normals
The direction ratios of the normals to the two planes are
a1 , b1 , c1 ; a2 , b2 , c2
If θ is the angle between the planes, then
a1 a2 + b1b2 + c1 c2
cos θ =
∑ a12 ∑ a22
Example 5.12
Find the angle between the planes.
2 x − y + z = 6; x + y + 2 z = 3
Solution
The angle between the planes is the angle between the normals to the planes. The
direction ratios of the normals are
(2, − 1, 1) and (1, 1, 2).
If θ is the angle between the planes, then
2 ×1 − 1×1 + 1× 2 3 1
cos θ = = =
6 6 6 2

∴ θ = 60o
Corollary 13
(a) The planes are parallel if
a1 b1 c1
= =
a2 b2 c2
(b) The planes are perpendicular if
a1 a2 + b1 b2 + c1 c2 = 0
Corollary 14
Angle between the line and a plane is the complement of the angle between the
normal to the plan and the line. Thus if θ is the angle between the planes
ax + by + cz + d = 0 and the line with direction ratios as l1 , m1 , n1 then (90o − θ)
is the angle between the normal and the line.
a l1 + b m1 + c n1
∴ cos (90o − θ) =
∑ a2 ∑ l12
Example 5.13
Find the equation of a plane through the point (1, 1, 3) and parallel to the plane
3x + 4 y − 5 z = 0 .
Solution
Let the equation of the plane parallel to the given plane be
3x + 4 y − 5 z + k = 0
As this plane passes through the point (1, 1, 3), we get
3 + 4 − 15 + k = 0 67
Mathematics-I This gives k = 8. Substituting this value, the required equation is
3x + 4 y − 5 z + 8 = 0
Example 5.14
Find the equation of the plane through the points (1, 2, − 3), (2, 3, − 4) and
perpendicular to the plane x + y + z + 1 = 0 .
Solution
The equation of the plane through the point (1, 2, − 3) is
a ( x − 1) + b ( y − 2) + c ( z + 3) = 0 . . . (1)
Since this plane passes through the point (2, 3, − 4).
∴ a+b−c=0
Now the plane (1) and the given plane x + y + z + 1 = 0 are perpendicular, we get
a × 1 + b × 1 + c × 1 = 0 or a + b + c = 0
Solving the two equations for a, b, c, we get
a b c
= =
1+1 −1−1 1−1
a b c
∴ = =
2 −2 0
Substituting for a, b, c in (1), we get
2 ( x − 1) − 2 ( y − 2) + 0 ( z + 3) = 0
or x − y +1= 0
This is the equation of the required plane.
5.3.5 Perpendicular Distance of a Point from a Plane
To find the perpendicular distance of a point ( x1 , y1 , z1 ) from the plane
ax + by + cz + d = 0
Let ABCD be the given plane ax + by + cz + d = 0 and PL the required perpendicular
distance of the point ( x1 , y1 , z1 ) from the plane.
P (x1, y1, z1)

D C

Q (0, 0, − d/c) L

A B

Figure 5.8

68
⎛ −d⎞ Three Dimensional
If c is not equal to zero, then a point Q ⎜ 0, 0, ⎟ will be point on the plane. The Geometry
⎝ c ⎠
direction ratios of the normal PL are a, b, c the coefficients of x, y, z in the equation of
a b c
the plane. The actual direction cosines of PL are , , .
∑ a2 ∑ a2 ∑ a2

From the figure PL is the projection of PQ on PL.


⎛ d⎞
a ( x1 − 0) + b ( y1 − 0) + c ⎜ z1 + ⎟
⎝ c ⎠ ax1 + by1 + cz1 + d
∴ PL = =
(a 2 + b 2 + c 2 ) (a 2 + b2 + c 2 )

This gives the required length of the perpendicular.


Note : Rule to find the perpendicular distance of a point from a plane (equation given in
general form).
In the left hand side of the equation of the plane (RHS being zero) substitute the
co-ordinates of the point and divide the result by

(coeff. of x) 2 + (coeff. of y ) 2 + (coeff. of z ) 2

The result gives the required perpendicular distance.


Example 5.14
The axes being rectangular, find the perpendicular distance of the origin from the
plane 2 x − y + 2 z − 6 = 0 .
Solution
Using perpendicular formula, the length of the normal from (0, 0, 0) is given by

−6 6
= = 2.
4 +1+ 4 3

Example 5.15
A corner between two walls at right angle is required to be filled in symmetrically
with earth so that the plane surface is 4 units from the point O on the ground line
where the wall meets. Find the distance from O to which the earth will extend
along the wall.
Solution
Take O as the origin. Let a be the intercepts of the plane on each axes, then the
equation of the plane is
x y z
+ + =1
a a a
or x+ y+z=a
Now we are given that the length of the perpendicular from (0, 0, 0) is 4 units
a
∴ =4
3

∴ a=4 3
Putting this value of a in the equation, we get
x+ y+z=4 3

∴ The required distance is 4 3 . 69


Mathematics-I SAQ 2
(a) Two systems of rectangle axes have the same origin. If a plane cuts them at
distances a, b, c and a1 , b1 , c1 from the origin, show that
1 1 1 1 1 1
2
+ 2
+ 2
= + +
a b c a12 b12 c12
(b) Find the length of the perpendicular from the point (1, 4, 5) on the plane
passing through the three points (2, − 1, 5), (0, − 4, 1) and (2, − 6, 0).
(c) The direction cosines of the perpendicular to the plane from the origin are
proportional to 1, 3, 1 and the length of the perpendicular is 2. Find the
equation of the plane.
(d) Find the equation of the plane through the points (2, 2, 1), (1, − 2, 3) and
parallel to x-axis.
(e) Find the equation of the plane passing through the points (2, 3, 4),
(− 3, 5, 1), (4, − 1, 2). Find also the angles which the normal to this plane
makes with the axes.
(f) Find the equation of the plane which passes through the intersection of the
planes 2 x + y − 4 = 0, y + 2 z = 0 and perpendicular to the plane
3x + 2 y − 3z = 6 .

(g) Find the equation of the plane that passes through the point (− 1, 1, − 4) and
is perpendicular to each of the planes.
− 2x + y + z + 2 = 0
x + y − 3z + 1 = 0

5.4 EQUATIONS OF A STRAIGHT LINE


(DEFINITION AND PROPERTIES)
5.4.1 General Form
Two planes, i.e. two equations of the first degree in x, y, z taken together define a straight
line.
Thus a1 x + b1 y + c1 z + d1 = 0 . . . (1)
and a2 x + b2 y + c2 z + d 2 = 0 . . . (2)

A B

Figure 5.9

together represent a straight line which is the line AB of intersection of the planes (1) and
(2).
70
5.4.2 Standard Forms Three Dimensional
Geometry
(a) Line passing through the point ( x1 , y1 , z1 ) and having its direction cosines l,
m, n is giving by
x − x1 y − y1 z − z1
= = =r . . . (3)
l m n
where ( x, y, z ) is any floating point on the line.
(b) Line passing through two points ( x1 , y1 , z1 ) and ( x2 , y2 , z2 ). The direction
ratios of the line are
x2 − x1 , y2 − y1 , z2 − z1
∴ The equation of the line is
x − x1 y − y1 z − z1
= = =r . . . (4)
x2 − x1 y2 − y1 z2 − z1

x − x2 y − y2 z − z2
or = = =r . . . (5)
x2 − x1 y2 − y1 z2 − z1

Forms (3), (4) and (5) are the standard equations of the straight line in
Symmetrical Form.
Important : Any point on the straight line is
x = x1 + l r ; y = y1 + m r ; z = z1 + n r

Corollary 15
To reduce the general equation of a straight line to symmetrical form
(a) find a point on the line by putting z = 0 in the two given equations and
solving the resulting equations for x and y.
(b) find the direction cosines of the line from the fact that the normals to the
planes will be perpendicular to the line because the line lies on both the
planes.
(c) having found out the point on the line and its direction cosines, write the
equation of the line in symmetrical form.
Example 5.16
Find in symmetrical form, the equations of the line
4 x + 4 y − 5 z = 12, 8 x + 12 y − 13 z = 32

(a) A point on the line, put z = 0 in the given equations, we get


4 x + 4 y − 12 = 0

8 x + 12 y − 32 = 0

or x+ y −3=0

2x + 3y − 8 = 0

x y 1
Solving = =
−8+9 −6+8 3−2

x y
or = =1
1 2
∴ A point on the line is (1, 2, 0). 71
Mathematics-I (b) Let l, m, n be the direction cosines of the line, then
∴ 4l + 4m − 5n = 0
8l + 12m − 32n = 0
l m n
Solving = =
2 3 4
∴ The equation in symmetrical form is
x −1 y − 2 z
= = .
2 3 4
5.4.3 Angle between the Line and a Plane
The angle between the line and a plane is the complement of the angle between the line
and the normal to the plane.
If θ is the angle between the line PQ and the plane ABCD, then 90o − θ is the angle
between the line PQ and normal PL to the plane ABCD.
L

D C
90 − θ

θ
M
P

A B

Figure 5.10
Example 5.17
x +1 y z − 3
Find the angle between the line = = and the plane 3x + y + z = 7 .
2 3 6
Solution
Let θ be the angle between the line and the plane, then 90o − θ is the angle
between the line and the normal to the plane. Direction ratios of line are 2, 3, 6 and
the direction ratios of the normal to the plane are 3, 1, 1.
2 × 3 + 3×1 + 6 ×1 15
∴ cos (90 − θ) = sin θ = =
(4 + 9 + 36) (9 + 1 + 1) 7 11
15
∴ θ = sin −1 .
7 11

5.5 BLOCK AND CUBE


In this section we study surfaces and solids and relate them with equations involving one
or more of the three variables x, y, z.
Case I
In the first case we discuss the locus of the equation f ( x) = 0 , i.e. the equation
involving one variable. It will be assumed that f ( x ) is a polynomial, i.e. an
expression involving positive integral powers of x only.
72
Let x1 , x2 , x3 , . . . , xk be the roots of f ( x) = 0 . Three Dimensional
Geometry
Then this equation can be written in the equivalent form as
f ( x ) = ( x − x1 ) ( x − x2 ) . . . ( x − xk ) = 0 . . . (1)
This will give
x − x1 = 0, x − x2 = 0 . . . x − xk = 0 . . . (2)
This shows that the co-ordinates of the points which satisfy (1) satisfy one or the
other of the equations (2).
∴ The points on the equation f ( x) = 0 lie on the one or the other of the planes.
x − x1 ; x − x2 ; . . . x − xk
which by definition are the planes parallel to yz-plane. Hence f ( x ) = 0 represents
a system of planes parallel to yz-plane.
Note : f ( y ) = 0 represents a system of planes parallel to zx-plane and f ( z ) = 0
represents a system of planes parallel to xy-plane.
Case II : Equation Involving Two Variables
We now discuss the locus of the equation f ( x, y ) = 0 .
The given equation is
f ( x, y ) = 0 . . . (3)
It will be assumed that f ( x, y ) = 0 is a polynomial in x and y.

Let Γ be the curve in the xy-plane and let its equation in the plane be
f ( x, y ) = 0 . . . (4)
Take any point P on this curve and let its co-ordinates be (x1, z1). Then from (4) we
have
f ( x1 , y1 ) = 0 . . . (5)

X
O

Y Γ
P

Figure 5.11

From P draw a straight line PQ parallel with the z-axis and let Q ( x1 , y1 , z ) be a
point on this line.
Substituting the co-ordinates of Q in (4), we get f ( x1 , y1 ) = 0 which is the same
as (5) above.
∴ Q lies on the locus of (4), i.e. any point on the line through P parallel with z-
axis and, therefore, the line PQ itself lies on the locus of (4).
73
Mathematics-I Thus (4) represents a cylinder generated by a straight line which is parallel to
the z-axis and intersects a curve whose equation in the xy-plane is f (x, y) = 0.
Note : f ( x, y ) = 0 represents a cylinder generated by a straight line which is
parallel with the z-axis and intersects a curve whose equation in the
yz-plane is
f ( x, y ) = 0 and f ( z , x) = 0
represents a cylinder generated by a straight which is parallel with the
y-axis and intersects a curve whose equation in the zx-plane is
f ( z, x) = 0 .
Case III : Equation Involving Three Variables
We now discuss the locus of the equation f ( x, y, z ) = 0 .
The equation of the surface is
f ( x, y , z ) = 0 . . . (6)
It will be assumed that f ( x, y , z ) is a polynomial in x, y, z.
Z

Q (x1, y1, z1)

M
X
O

(x1, y1, 0)
Y P

Figure 5.12
Take a point P in the xy-plane and let its co-ordinates be (x1, y1).
Through P draw a line PQ parallel to the z-axis and let Q ( x1 , y1 , z ) be any point
on this line PQ. If Q ( x1 , y1 , z ) lies on the locus of (6), then
f ( x1 , y1 , z ) = 0
This is an equation in z, having a finite number of roots.
Therefore a line through any point P in the xy-plane parallel to z-axis meets the
locus of (6) in a finite number of points.
Thus (6) represents a surface.
To Sum Up, We Say that
(a) f ( x) = 0 represents a system of planes parallel to yz-plane.
(b) f ( x, y ) = 0 represents a cylinder generated by a straight line which is
parallel to the z-axis.
(c) f ( x, y, z ) = 0 represents a surface.
Example 5.18
What surfaces are represented by
x2 y2
(i) − =1
a2 b2
(ii) y 2 = 4 ax
74
Solution Three Dimensional
Geometry
Since (i) and (ii) are of the type f ( x, y ) = 0 .
(i) represents a cylinder generated by a straight line which is parallel to the
z-axis and intersects a hyperbola whose equation in the xy-plane is
x2 y2
− =1 (It is a hyperbolic cylinder).
a2 b2
(ii) represents a cylinder generated by a straight line which is parallel to z-axis
and intersects a curve whose equation in the xy-plane is y 2 = 4ax . This
cylinder is called the parabolic cylinder.
Important : Short notes on some important surfaces.
Tetrahedron
Definition
(a) A Tetrahedron is a pyramid whose base is a triangle. It is a figure
bounded by four planes and thus ABCD is defined as a tetrahedron
(tetra means four and a tetrahedron, four faced figure).
A

C D

Figure 5.13

(b) Vertices : Since three planes meet in a point, a tetrahedron has four
vertices, the points A, B, C, D.
(c) Faces : It has four triangular faces, namely the triangles ABC, ABD,
ACD, BCD.
(d) Edges : Since two planes meet in a line, a tetrahedron has (4C 2 = 6)
6 edges, namely AB, AC, AD, BC, BD, CD.
Opposite Edges : It has three pair of opposite edges which do not
meet namely AB and CD; AD and BC; AC and BD.
(e) Centre of Gravity : If ABCD is the tetrahedron and G1 is the centre
of gravity of the base triangle BCD, then G, the centre of gravity of
the tetrahedron divides AG1 in the ratio of 3 : 1.
Centre of Gravity Formula
If the four vertices of the tetrahedron have their co-ordinates
A = ( x1 , y1 , z1 ), B ( x2 , y2 , z2 ), C ( x3 , y3 , z3 ), D ( x4 , y4 , z4 ) ,
then the co-ordinates of the centre of gravity G of the
tetrahedron are given by
x1 + x2 + x3 + x4 y + y2 + y3 + y4 z + z + z 3 + z4
x= ;y= 1 ;z= 1 2
4 4 4
75
Example 5.19
Mathematics-I Show that if two pairs of opposite edges of a tetrahedron are perpendicular, then
the third pair is also perpendicular.
Solution
Let ABCD be a tetrahedron. Let the point A be supposed to coincide with the origin
and the other vertices B, C and D have their co-ordinates
( x1 , y1 , z1 ), ( x2 , y2 , z2 ), ( x3 , y3 , z3 ) .
Suppose AB is perpendicular to CD and AC is perpendicular to BD.
We have to prove that AD is perpendicular to BC. The direction ratio of AB are
x1 − 0, y1 − 0, z1 − 0; i.e. x1 , y1 , z1

The directions of CD are x2 − x3 , y2 − y3 , z2 − z3 .


Since the two lines are perpendicular, we have
x1 ( x2 − x3 ) + y1 ( y2 − y3 ) + z1 ( z2 − z3 ) = 0
Similarly as AC is perpendicular to BD, we get
x2 ( x3 − x1 ) + y2 ( y3 − y1 ) + z2 ( z3 − z1 ) = 0
Adding the above two equations, we have
x3 ( x2 − x1 ) + y3 ( y2 − y1 ) + z3 ( z2 − z1 ) = 0
Showing there by that AD is perpendicular to BC.
Parallelopiped
Definition
(a) A parallelopiped is a figure bounded by three pairs of parallel planes
(i.e. the parallelograms). Thus OBLCMANP is a parallelopiped.
C M

L
P

A
O

B N

Figure 5.14

(b) Faces : It has six faces viz., the parallelograms OBLC; MANP;
LCMP; OANB; OCMA; NBLP.
(c) Diagonals : It has four diagonals, viz., the four lines OP, BM, AL and
NC.
A Rectangular Parallelopiped
If all the six faces in a parallelopiped are rectangles, then it is defined as a
rectangular parallelopiped.
A Cube
If all the six faces in a parallelopiped are squares then it is defined as a cube.
76
Example 5.20 Three Dimensional
Geometry
Find the angle between any two diagonals of a cube.
Solution
Let O, one vertex of a cube be taken as the origin of co-ordinates and the three
edges through O as the axes of co-ordinates. In a cube, all planes are square. Let a
be the length of one edge and we take two diagonals as OP and AL. The
co-ordinates of four points are
O (0, 0, 0), A (a, 0,0), L (0, a, a ), P (a, a, a )
The direction cosines of OP and AL are
1 1 1 1 1 1
, , ; − , ,
3 3 3 3 3 3
If θ is the angle between two diagonals, then
−1+1+1 1
cos θ = =
3 3
⎛1⎞
∴ θ = cos −1 ⎜ ⎟ .
⎝3⎠
Example 5.21
A line makes angles α, β, γ, δ with the four diagonals of a cube, prove that
4
cos 2 α + cos 2 β+ cos 2 γ + cos 2 δ = .
3
Solution
Take O, one vertex of the cube, as origin and three edges through O as
co-ordinates axes. (See Figure 5.14).
Let a be the length of an edge of a cube. The co-ordinates of the end points of the
diagonals are O (0, 0, 0), A (a, 0,0), B (0, a, 0), C (0, 0, a ), L (0, a, a),
N (a, a, 0), M (a, 0, a) and P (a, a, a) .
The direction cosines of OP, AL, BM and CN are
1 1 1 1 1 1 1 1 1 1 1 1
, , ;− , , ; ,− , ; , ,−
3 3 3 3 3 3 3 3 3 3 3 3
If l, m, n are direction cosines of the given line then
l+m+n −l +m+n
cos α = cos β =
3 3
l−m+n l+m−n
cos γ = cos δ =
3 3
Squaring and adding, we get
cos 2 α + cos 2 β + cos 2 γ + cos 2 δ
1
= [(l + m + n) 2 + (− l + m + n) 2 + (l − m + n) 2 + (l + m − n)2 ]
3
1 4 4
= (4l 2 + 4m 2 + 4n 2 ) = (l 2 + m2 + n 2 ) =
3 3 3
Example 5.22
A (6, − 16, − 3), B (3, − 1, 4), C (− 2, 5, 0) are the vertices of a triangle. Find
co-ordinates of the foot of the perpendicular drawn from A to BC.
Solution 77
Mathematics-I Let D be the foot of the perpendicular drawn from A to BC. If D divides BC in the
⎛ − 2k + 3 5k − 1 4 ⎞
ratio of k : 1, its co-ordinates are ⎜ , , ⎟ . The direction ratios
⎝ k +1 k + 1 k + 1⎠
⎛ − 2k + 3 5k − 1 4 ⎞
of AD are ⎜ − 6, + 16, + 3⎟ .
⎝ k +1 k +1 k +1 ⎠
− 8k − 3 21k + 15 3k + 7
i.e. , ,
k +1 k +1 k +1
A

B C
D
Figure 5.15

The direction ratios of BC are (5, − 6, 4).


Since AD is perpendicular to BC
∴ 5 (− 8k − 3) − 6 (21k + 15) + 4 (3k + 7) = 0
1
k=−
2
The co-ordinates of D are (8, − 7, 8).
SAQ 3
(a) Two edges AB ard CD of a tetrahedron are perpendicular. Prove that the
distance between the mid-points of AC and BD is equal to the distance
between the mid-points of AD and BC.
(b) Show that the points (1, 2, 3), (− 1, − 2, − 1), (2, 3, 2), (4, 7, 6) are the
vertices of a parallelogram.
(c) Show that the four points (1, 1, 1), (− 2, 4, 1), (− 1, 5, 5), (2, 2, 5) are the
vertices of a square.
(d) If the edges of a rectangular parallelopiped are a, b, c show that the angles
between four diagonals are given by
a 2 ± b2 ± c 2
cos −1
a 2 + b2 + c 2
(e) AB, BC, CA are the diagonals of adjacent faces of a rectangular box with its
centre at the origin O, its edges parallel to axes. If the angles BOC, COA,
AOB are equal to θ, φ, ψ respectively, prove that
cos θ + cos φ + cos ψ = − 1 .

78
Three Dimensional
5.6 SPHERE, CONE AND CYLINDERS Geometry

5.6.1 Sphere
Definition
The locus of a point which moves in space such that it remains at a constant
distance from a fixed point is defined as a spherical surface or a sphere. The fixed
point is called the centre of the sphere and the fixed distance is called the radius. In
spherical geometry ‘sphere’ means a hollow sphere of negligible thickness.
Standard Form of the Equation of the Sphere
To find the equation of the sphere whose centre is the point (a, b, c) and whose
radius is r.

P (x, y, z)

A B
C (a, b, c)

Figure 5.16
Let P ( x, y , z ) be any point on the sphere and r its radius. Let C (a, b, c ) be the
centre of the sphere.
By definition and by distance formula
( x − a ) 2 + ( y − b) 2 + ( z − c ) 2 = r 2 . . . (1)
Relation (1) being true for every point on the sphere represents the equation of the
sphere in Centre and Radius form or more generally the central form of the
equation of the sphere. If the centre of the sphere coincides with the origin of
co-ordinates, i.e. a = 0, b = 0, c = 0, then the equation of the sphere is given by
x2 + y2 + z 2 = r 2
This equation of the sphere is called the Standard Form of the equation of the
sphere.
Others Forms of the Equation of the Sphere
General Form
To show that the equation
x 2 + y 2 + z 2 + 2ux + 2vy + 2 wz + d = 0
represents a sphere whose centre is (− u , − v, − w) and whose radius is
u 2 + v 2 + w2 − d .

The given equation can be written as

( x + u ) 2 + ( y + v ) 2 + ( z + w) 2 = u 2 + v 2 + w2 − d . . . (2)

79
Mathematics-I This shows that the sum of the square of the distance of any point (x, y, z) on
the sphere from the point (− u , − v, − w) is constant and is equal to
u 2 + v 2 + w2 − d .
Hence the Eq. (2) represents a sphere whose centre is the point
(– u, – v, – w) and whose radius is u 2 + v 2 + w2 − d .
This equation is called the General Form of the equation of the sphere.
Note : Like the equation of the circle in plane co-ordinate geometry, the
equation of the sphere has three characteristics.
(a) It is a second degree equation in x, y, z.
(b) The coefficients x 2 , y 2 , z 2 are all equal.
(c) The product terms xy, yz, zx are absent.
Corollary 16
General rule to write the co-ordinates of the centre of the sphere and the
length of its radius.
If the coefficient of x 2 , y 2 , z 2 are each equal to unity, then the co-ordinates of the
1 1 1
centre are ( − the coeff. of x, − the coeff. of y, − the coeff. of z).
2 2 2
The square of the radius is equal to the sum of the squares of the
co-ordinates of the centre minus the constant term.
Intercept Form
To find the equation of the sphere which cuts off intercepts a, b, c on the co-
ordinates axes and also passes through the origin, i.e. (0, 0, 0).
Consider the equation of the sphere
x 2 + y 2 + z 2 + 2ux + 2vy + 2 wz + d = 0 . . . (3)
Since the sphere passes through the origin, putting x = 0, y = 0, z = 0 in the
above equation, we get d = 0.
Equation (3) takes the form
x 2 + y 2 + z 2 + 2ux + 2vy + 2 wz = 0
This gives the general equation of the sphere through the origin.
Q The sphere cuts off intercept a on the x-axis.
∴ The point (a, 0, 0) lies on it. This gives
a
a 2 + 2ua = 0 or u = −
2
b c
Similarly for the other two intercepts v = − and w = − .
2 2
Putting the values of u, v, w in the above equation, we obtain the equation of
the sphere in the intercept form as
x 2 + y 2 + z 2 − ax − by − cz = 0
Diameter Form
To find the equation of the sphere on the join of ( x1 , y1 , z1 ) and ( x2 , y2 , z2 )
as extremities of the diameter.
80
Let A, B be the points ( x1 , y1 , z1 ) and ( x2 , y2 , z2 ) as extremities of the Three Dimensional
Geometry
diameter. Let P ( x, y , z ) be any point on the sphere. (Refer Figure 5.1).
Join PA and PB.
Then ∠ APB = 90o (Angle in the semi-circle).
Direction ratio of PA and PB are
( x − x1 , y − y1 , z − z1 )
and ( x − x2 , y − y2 , z − z2 ) .
Since PA and PB are perpendicular, using the condition of perpendicularity,
we get
( x − x1 ) ( x − x2 ) + ( y − y1 ) ( y − y2 ) + ( z − z1 ) ( z − z2 ) = 0
This is the equation of the sphere in diameter form.
Equation of the Sphere through Four Points
The general equation of the sphere contains four arbitrary constants
u, v, w, d. Hence to determine a sphere completely, we need to have four
conditions, each of which will give rise to one independent equation in the
condition. As a case, we pass a sphere through four points and derive its
equation.
Let the equation of the sphere be
x 2 + y 2 + z 2 + 2ux + 2vy + 2 wz + d = 0
and the four points through which it passes be
( x1 , y1 , z1 ), ( x2 , y2 , z2 ), ( x3 , y3 , z3 ), ( x4 , y4 , z4 )
Substituting in the equation, we get
x12 + y12 + z12 + 2ux1 + 2vy1 + 2 wz1 + d = 0

x22 + y22 + z22 + 2ux2 + 2vy2 + 2 wz2 + d = 0

x32 + y32 + z32 + 2ux3 + 2vy3 + 2 wz3 + d = 0

x42 + y42 + z42 + 2ux4 + 2vy4 + 2 wz4 + d = 0


Eliminating u, v, w and d from the above equations, we get
x2 y2 z2 x y z 1
x12 y12 z12 x1 y1 z1 1
x22 y22 z22 x2 y2 z2 1 = 0
x32 y32 z32 x3 y3 z3 1
x42 y42 z42 x4 y4 z4 1

Expanding the determinant, we get the desired equation.


Example 5.23
Find the equation of the sphere which passes through the points (a, 0, 0), (0, a, 0),
(0, 0, a) and whose centre lies on the plane x + y + z = a .
Solution
Let the equation of the sphere be

x 2 + y 2 + z 2 + 2ux + 2vy + 2 wz + d = 0
81
Mathematics-I Substituting the points (a, 0, 0), (0, a, 0) and (0, 0, a), we get

a 2 + 2ua + d = 0

a 2 + 2va + d = 0

a 2 + 2 wa + d = 0
Solving these equations, we get
u=v=w
The co-ordinates of the centre of the sphere are (− u , − v, − w) . Since the centre
lies on the plane x + y + z = a , it must satisfy the equation, this gives
− u − v − w = a or u + v + w = − a

Since u = v = w, we get
a
3u = − a or u = − =v=w
3
a
Putting u = − in the first of the above three equations, we obtain
3
a2
d =−
3
Substituting the values of u, v, w, d in the equation of the sphere, we get

3 ( x 2 + y 2 + z 2 ) − 2a ( x + y + z ) − a 2 = 0
which is the required equation of the sphere.
Example 5.24
A plane passes through a fixed point (a, b, c) and cuts the axes in A, B, C. Show
that the locus of the centre of the sphere OABC is
a b c
+ + =2
x y z

Solution
The equation of the sphere (Refer Section 5.6.1) is

x 2 + y 2 + z 2 − α x − β y − γz = 0

where A, B, C are the points (α, 0, 0), (0, β, 0) (0, 0, γ).


The equation of the plane through the points A, B, C is
x y z
+ + =1
α β γ
The plane passes through (a, b, c). This gives
a b c
+ + =1.
α β γ
The co-ordinates of the centre of the sphere are
⎛α β γ⎞
⎜ , , ⎟
⎝ 2 2 2⎠

82
α β γ Three Dimensional
Let = X , α = 2X ; = Y , β = 2Y ; = Z , γ = 2Z Geometry
2 2 2
Substituting for α, β, γ in terms of X, Y, Z, the locus of the centre of the sphere is
a b c
+ + =1
2 X 2Y 2 Z
a b c
or + + = 2.
x y z
Example 5.25
Find the equation of the sphere, which passes through the points (1, 0, 0), (0, 2, 0),
(0, 0, 3) and has its radius as small as possible.
Solution
Let the equation of the sphere be

x 2 + y 2 + z 2 + 2ux + 2vy + 2 wz + λ = 0
Passing the sphere through the three points, we get
1+ λ
1 + 2u + λ = 0 ; u = −
2
4+λ
4 + 4v + λ = 0 ; v = −
4
9+λ
9 + 6w + λ = 0 ; w = −
6
The radius of the sphere is
(1 + λ ) 2 (4 + λ )2 (9 + λ ) 2
R 2 = u 2 + v 2 + w2 − λ = + + −λ
4 16 36
36 (1 + λ ) 2 + 9 (4 + λ ) 2 + 4 (9 + λ ) 2 − 144 λ
=
144
49 λ 2 + 72 λ + 504
=
144
dR
For radius to be as small as possible, we apply the condition = 0.

dR
This gives 2R = 98 λ + 72

36
or 98 λ + 72 = 0 or λ = −
49
Substituting for λ in u, v, and w, we get
13 40 135
u=− ;v=− ;w=−
98 49 98
Replacing the values of u, v, w in the equation of the sphere, we obtain the desired
equation as
13x 80 y 135 z 36
x2 + y 2 + z 2 − − − − =0
49 49 49 49
or 49 ( x 2 + y 2 + z 2 ) − (13 x + 80 y + 135 z ) = 36
83
Mathematics-I Example 5.26
Find the equation of the sphere which touches the co-ordinate axes, whose centre
is in the positive octant and has radius 4.
Solution
Let the equation of the sphere be

x 2 + y 2 + z 2 + 2ux + 2vy + 2 wz + d = 0

The sphere touches the x-axis. The direction cosines of x-axis are (1, 0, 0).
Q The equation of x-axis is
x y z
= = = λ (say)
1 0 0
∴ x = λ, y = 0, z = 0
Putting these values of x, y, z in the equation of the sphere, we get

λ 2 + 2uλ + d = 0

This is a quadratic equation in λ, has two roots where x-axis cuts the sphere. But
x-axis touches the surface and therefore both the roots must be equal, condition for
which is

4u 2 − 4d = 0 or u 2 = d
Similarly for y and z-axes we get v2 = d and w2 = d as both y- and z-axes also touch
the sphere.
If r is the radius of the sphere

r = [u 2 + v 2 + w2 − d ] = (3d − d ) = 2d = 4 (given)

∴ 2d = 16 or d = 8

Again u 2 = v 2 = w2 = 8

∴ u=v=w=±2 2
Center of the sphere is (− u , − v, − w) .
Since the center lies in the positive octant, we take
u=v=w=−2 2

∴ Centre is the point (2 2, 2 2, 2 2) .


Substituting for u, v, w in the equation of the sphere, we get

x2 + y2 + z 2 − 4 2 ( x + y + z) + 8 = 0

Example 5.27
Find the equation of the sphere passing through the points (0, 0, 0); (0, 1, − 1);
(− 1, 2, 0); (1, 2, 3).
Solution
Let the required equation of the sphere be

x 2 + y 2 + z 2 + 2ux + 2vy + 2 wz + d = 0
84
Substituting the points in the given equation, we get Three Dimensional
Geometry
d=0
2 + 2v − 2 w + d = 0

5 − 2u + 4v + d = 0

14 + 2u + 4v + 6w + d = 0

or 2v − 2w + 2 = 0 . . . (i)

− 2u + 4v + 5 = 0 ...
(ii)
2u + 4v + 6w + 14 = 0 . . . (iii)

Eliminating w from the first and the third equations, we get


u + 5v + 10 = 0

and 2u − 4v − 5 = 0

Solving by cross multiplication, we get


u v 1
= =
− 25 + 40 20 + 5 − 4 − 10

u v 1
or = =
15 25 − 14

15 25 11
∴ u=− , v = − , w = v +1= −
14 14 14
Substituting for u, v, w, and d, the equation of the sphere is given by

7 ( x 2 + y 2 + z 2 ) − 15 x − 25 y −11z = 0

Plane Section of a Sphere


We know that the section of a sphere by a plane is a circle.

L P
Q

Figure 5.17

If the plane passes through the centre of a sphere, then the section of the
sphere is known as great circle. The centre and the radius of the great circle
are the same as the centre and the radius of the sphere.

85
Mathematics-I Let PQR be the plane cutting the sphere in a circle whose centre is L and
whose radius is PL. Now
(a) the line joining the centre of the sphere with the centre of the section
is perpendicular to the plane of the section. In other words the line CL
is at right angle to the plane PQR.
(b) if r is the radius of the section and R that of the sphere, then

r 2 = R 2 − CL2
where CL is the length of the perpendicular from the centre of the
sphere on the plane PQR.
If the equation of the plane and the sphere are given, the co-ordinates
of the centre of the section will be obtained by the point of
intersection of the plane and the normal to the plane from the centre
of the sphere.
Having found out the radius of the section the area of the section can
be conveniently calculated.
Important : Equation of a Circle
A circle will be represented by two equations, one that of a sphere and the other
of a plane taken together.
Example 5.28
Deduce the equation of the sphere despoiled on the line joining the point
(2, − 1, 4) and (− 2, 2, − 2) as the end points of the diameter. Find also the area of
the circle in which the sphere is intercepted by the plane.
2x + y − z = 3
Solution
The equation of the sphere is
( x − 2) ( x + 2) + ( y + 1) ( y − 2) + ( z − 4) ( z + 2) = 0

or x 2 + y 2 + z 2 − y − 2 z − 14 = 0

⎛ 1 ⎞ 61
The co-ordinates of the centre of the sphere are ⎜ 0, , 1⎟ and its radius is .
⎝ 2 ⎠ 2
If p is the length of the perpendicular from the centre of the sphere, i.e. from
⎛ 1 ⎞
⎜ 0, , 1⎟ on the plane 2 x + y − z − 3 = 0 , then
⎝ 2 ⎠
1
−1− 3
7
p= 2 = (in magnitude)
6 2 6
If r is the radius of the circle, then
61 49 366 − 49 317
r 2 = R2 − p2 = − = =
4 24 24 24
Now the area of the circle is
317
πr2 = π
24
Example 5.29
86
Obtain the equation of the sphere having the circle Three Dimensional
Geometry
x 2 + y 2 + z 2 + 10 y − 4 z − 8 = 0 ; x + y + z = 3 as a great circle.

87
Solution Three Dimensional
Geometry
The equation of any sphere passing through the given circle is
x 2 + y 2 + z 2 + 10 y − 4 z − 8 + k ( x + y + z − 3) = 0

or x 2 + y 2 + z 2 + kx + (10 + k ) y + ( k − 4) z − 8 − 3 k = 0
The centre of this sphere is
⎛ k 10 + k (k − 4) ⎞
⎜− , − ,− ⎟
⎝ 2 2 2 ⎠
If the given circle is great circle, then the centre of the sphere must lie on the plane
x+ y + z −3=0
k 10 + k k − 4
∴ − − − −3=0
2 2 2
Solving for k, we get
k=−4
Putting the value k in the equation of the sphere we get
x 2 + y 2 + z 2 + 10 y − 4 z − 8 − 4 ( x + y + z − 3) = 0

or x2 + y 2 + z 2 − 4 x + 6 y − 8z + 4 = 0
This is the required equation of the sphere.
SAQ 4
(a) Find the centre and radius of the sphere given by
x2 + y 2 + z 2 + 2 y − 4z = 4
(b) Find the centre and the radius of the circle in which the sphere
x 2 + y 2 + z 2 + 2 x − 2 y − 4 z − 19 = 0 is cut by the plane
x + 2 y + 2 z + 7 = 0 . Also find the equation of the sphere having the above
circle as a great circle.
(c) Find the equation of the sphere which passes through the circle
x 2 + y 2 = 4, z = 0 and is cut by the plane x + 2 y + 2 z = 0 in a circle of
radius 3.
(d) Prove that the plane x + 2 y − z = 3 cuts the sphere
x 2 + y 2 + z 2 − x − z − 2 = 0 in a circle of radius unity.
(e) Find the equation of the sphere through the circle
x 2 + y 2 + z 2 = 9; 2 x + 3 y + 4 z = 5 and the point (1, 2, 3). Locate its centre
and find its radius.
(f) Find the equation of a sphere whose radius is 4 and whose centre is the point
(− 6, 1, 3). Find also the area of the section in which the sphere is cut by the
plane x − y + 2 z + 5 = 0 .
(g) Find the equation of the sphere passing through the points (1, 0, − 1);
(2, 1, 0); (1, 1, – 1) and (1, 1, 1).
(h) Find the equation of the sphere which passes through the point (3, 1, 2) and
meets the XOY plane in a circle of radius 3 units with the centre (1, − 2, 0).
x −1 y + 2 z − 0
(Hint : Equation of the line = = = k (say)).
0 0 1
87
Mathematics-I 5.6.2 Cone
Definition
A cone is defined as a surface generated by a straight line which passes through a
fixed point, and which satisfies one more condition, e.g. it may intersect a fixed
curve or touch a given surface.
(i) the straight line in any position is called a generator.
(ii) The fixed point is called the vertex.
(iii) the fixed curve is called the guiding curve of the cone.
Any cone whose vertex is the point at origin, will be represented by a
homogeneous equation in x, y, z, i.e. all the terms in the equation will be of the
same degree. This is due to the fact that every generator will pass through the
origin and its equation will be of the type :
x y z
= = =k
l m n
such that x = l k, y = m k, z = n k
This shows that the point (lk, mk, nk) will satisfy the equation of the cone for every
value of k. This will happen only if the equation is homogenous. Thus
x 2 + y 2 = z 2 ; x 2 = y z; 3 x 2 + 5 y 2 + 7 z 2 = 0
all represent cones.
Example 5.30
Find the equation of the cone whose vertex is the origin and guiding curve the
circle
x2 + y 2 + z 2 + 4 x + 2 y − 6 z + 5 = 0
2x + y + 2z + 5 = 0
Solution
The equation of any line through the origin is
x y z
= =
l m n
Any point on the line is (lk, mk, nk).
If this point lies on the circle given by the above equations, then
(l 2 + m 2 + n 2 ) k 2 + (4l + 2 m − 6 n ) k + 5 = 0
and (2l + m + 2 n ) k + 5 = 0
Eliminating k from these two equations, we get
25 5
(l 2 + m 2 + n 2 ) − (4l + 2 m − 6 n ) +5=0
(2l + m + 2 n ) 2 (2l + m + 2 n )

or 5 (l 2 + m 2 + n 2 ) − (4l + 2 m − 6 n ) (2l + m + 2 n ) + (2l + m + 2 n ) 2 = 0

or (l 2 + 4 m 2 + 21n 2 ) − 4lm + 6 mn + 12 nl = 0
Eliminating l, m, n from the equation with the help of the equation of the straight
line, the required equation of the cone is
x 2 + 4 y 2 + 21z 2 − 4 xy + 6 yz + 12 zx = 0

88
Note : This equation of the cone with its vertex at the origin is a homogeneous Three Dimensional
Geometry
equation of the second degree.
Rule
To find the equation of the cone whose vertex is the origin and the guiding
curve represented by two equations, we make one of two equations
homogeneous with the help of the other.
Example 5.31
Find the equation of the cone which has its vertex at the origin and base as the
circle x = a , y 2 + z 2 = b 2 . Show also that the section of the cone by a plane
parallel to the XOY plane is a hyperbola.
Solution
The required equation will be homogeneous of the second degree in x, y, z because
the cone has its vertex lying at the origin. Now
x
=1
a

Making y 2 + z 2 = b 2 homogeneous with the help of above relation, we get


2
⎛x⎞
y 2 + z 2 = b2 ⎜ ⎟
⎝a⎠

or a2 ( y 2 + z 2 ) = b2 x2

This is the required equation of the cone.


Any plane parallel to the XOY-plane is z = k .

Putting z = k in the equation of the cone, we get

a2 y 2 + a2 k 2 = b2 x2

or b2 x2 − a2 y 2 = a2 k 2

This is the equation of a hyperbola in the plane z = k .

Right Circular Cone


Definition
A right circular cone is the surface generated by a straight line which passes
through a fixed point and makes a constant angle with a fixed line through the
fixed point
(a) the fixed point is called the vertex.
(b) the fixed line is called the axis.
(c) the constant angle is called semi-vertical angle of the cone.
The section of a right circular cone by any plane perpendicular to its axis is a circle.
Note : Why the Name Right Circular Cone.
Since the section of a cone, generated by a straight line which passes through a fixed
point and makes a constant angle with a fixed line through a fixed point, by any plane at
right angles to the fixed line is a circle, hence the name ‘right circular cone’.
Standard Form of the Equation of a Right Circular Cone 89
Mathematics-I Find the equation of a right circular cone whose vertex is the origin, axis the z-axis
and semi-vertical angle α.

P (x, y, z)

X
O (0, 0, 0)

Y
Figure 5.18
Let P ( x, y , z ) be any point on the cone, so that ∠ ZOP = α .
∴ tan ZOP = tan α
The direction cosines of OZ, i.e. z-axis are (0, 0, 1) and that of OP are
( x − 0, y − 0, z − 0) i.e. ( x , y , z )

sin α 1 − cos 2 α
Now tan ZOP = tan α = =
cos α cos α
0 . x + 0 . y + 1. z z
But cos α = =
(x + y + z )
2 2 2
(x + y2 + z2 )
2

( x2 + y 2 )
∴ tan α =
z
Squaring, we get
x 2 + y 2 = z 2 tan 2 α
This is the required equation of the right circular cone in standard form as this is
the most simplest form of the equation.
Example 5.32
Find the equation of the right circular cone whose vertex is at the origin, whose
x y z
axis is the line = = and which has a semi-vertical angle 30o.
1 2 3
Solution
Let P ( x, y , z ) be any point on the cone. Direction ratios of OP are x, y, z.
Direction ratios of the line which is the axis through the vertex are 1, 2, 3.
x + 2 y + 3z 3
∴ cos 30o = =
( x + y + x ) 14
2 2 2 2

On squaring and simplifying, we get


19 x 2 + 13 y 2 + 3 z 2 − 8 xy − 24 yz − 12 zx = 0
Example 5.33
90
Find the semi-vertical angle and the equation of the right circular cone having its Three Dimensional
Geometry
vertex at the origin and passing through the circle y 2 + z 2 = 25, x = 4 .

Solution
The axis of the cone is a line through the vertex (0, 0, 0) and perpendicular to the
plane of the given circle, i.e. perpendicular to the plane x = 4. Hence x-axis is the
axis of the cone and its direction cosines are (1, 0, 0).

Putting y = 0 in y 2 + z 2 = 25 , we get z = 5.

Therefore, the circle passes through the point (4, 0, 5). Direction ratios of the
generator passing through the point (0, 0, 0) and (4, 0, 5) are 4, 0, 5.
∴ Direction cosines of the generator are
4 0 5
, ,
41 41 41
4
If the semi-vertical angle is α, then cos α = .
41
4
∴ α = cos −1
41

The given base is y 2 + z 2 = 25, x = 4 .

The equation of the cone whose vertex is the origin is a homogeneous equation of
second degree. Making equation y 2 + z 2 = 25 homogeneous with the help of
x = 4, we get
2
⎛ x⎞
y + z = 25 ⎜ ⎟
2 2
⎝4⎠

or 25 x 2 − 16 y 2 − 16 z 2 = 0

This is the required equation of the cone.


SAQ 5
(a) Find the equation of the cone whose vertex is the point (3, 1, 2) and whose
generating lines pass through the ellipse 2 x 2 + 3 y 2 = 1, z = 0 .

(b) Find the equation of the cone whose vertex is the point (1, 1, 0) and whose
guiding curve is x 2 + z 2 = 4, y = 0 .

(c) Find the equation of the cone whose vertex is (1, 2, 3) and which has the
guiding curve x 2 + y 2 + z 2 = 4, x + y + z = 1 .

(d) Find the equation of the cone with vertex at the origin and generators
touching the sphere x 2 + y 2 + z 2 − 2 x + 4 z = 1 .

(e) Find the equation of the conic surface whose vertex is the point P (0, 0, 1)
and whose guiding curve is

x2 y 2
+ = 1, z = 3
25 9
Obtain the section of the derived cone by the plane y = 0 and identify its
type.

91
Mathematics-I (f) Find the equation to the right circular cone whose vertex is P (2, − 3, 5)
and axis PQ which makes equal angles with the axes and which pass
through A (1, − 2, 3) .

(g) Find the equation of the right circular cone whose vertex is at the origin,
whose axis is the line
x y z
= =
1 2 3
and which has a semi-angle 30o.
(h) Find the equation of the right circular cone with its vertex (1, 2, 4) and
passing through the circle.

x 2 + y 2 + z 2 − 4 z = 11, x + 2 y + 2 z = 15

5.6.3 The Cylinder


Definition
A cylinder is defined as a surface generated by a straight line which is parallel to a
fixed line and satisfies one more condition, e.g. it may intersect a fixed curve.
(a) The straight line in any position is called a generator.
(b) The fixed curve is defined as the guiding curve of the cylinder.
Example 5.34
Find the equation of a cylinder whose generating lines are having their direction
cosines (l, m, n) and which pass through the circumference of the fixed circle
x 2 + z 2 = a 2 in the ZOX plane.
Solution
Let ( x1 , y1 , z1 ) be a point on the cylinder. Then the equations of the generator
through ( x1 , y1 , z1 ) are
x − x1 y − y1 z − z1
= = . . . (1)
l m n
The line will intersect the given curve
x2 + z2 = a2 ; y = 0
Putting y = 0, in Eq. (1), we get
l y1 n y1
x = x1 − ; z = z1 −
m m
Substituting the values of x, y, z in x 2 + z 2 = a 2 , we get
2 2
⎛ l y1 ⎞ ⎛ n y1 ⎞
⎜ x1 − ⎟ + ⎜ z1 − ⎟ =a
2
⎝ m ⎠ ⎝ m ⎠

or ( m x1 − l y1 ) 2 + ( m z1 − n y1 ) 2 = m 2 a 2

∴ The locus of ( x1 , y1 , z1 ) is the surface given by

(m x − l y )2 + (m z − n y )2 = m 2 a 2
92
Example 5.35 Three Dimensional
Geometry
Find the equation of the cylinder whose generators are parallel to the line
y z
x = − = and whose guiding curve is the ellipse x 2 + 2 y 2 = 1, z = 3 .
2 3
Solution
The direction ratios of the generators are (1, − 2, 3) . The equations of the
generator through the point ( x1 , y1 , z1 ) on the cylinder are
x − x1 y − y1 z − z1
= = . . . (2)
1 −2 3
This generator will intersect the curve.
Putting z = 3 in Eq. (2), we obtain
3 − z1 6 − 2 z1
x = x1 + ; y = y1 −
3 3
Substituting the value of x and y in x 2 + 2 y 2 = 1 , we have
2 2
⎛ 3 − z1 ⎞ ⎛ 6 − 2 z1 ⎞
⎜ x1 + ⎟ + 2 ⎜ y1 − ⎟ =1
⎝ 3 ⎠ ⎝ 3 ⎠
The locus of ( x1 , y1 , z1 ) is
2 2
⎛ 3− z⎞ ⎛ 6 − 2z ⎞
⎜x + ⎟ + 2⎜y − ⎟ =1.
⎝ 3 ⎠ ⎝ 3 ⎠
5.6.4 Right Circular Cylinder
Definition
A right circular cylinder is a surface generated by a straight line which is parallel
to a fixed line, and is at a constant distance from it.
(a) The fixed straight line is called the axis.
(b) The constant distance is called the radius of the cylinder.
Important
The section of a right circular cylinder by any plane perpendicular to its axis is a
circle.
Note : Why the name Right Circular Cylinder.
Since the section of a cylinder, generated by a straight line which is parallel to a
fixed line and is at a constant distance from it, by any plane at right angles to the
fixed line is a circle, hence the name right circular cylinder.
Example 5.36
Find the equation to the right circular cylinder of radius 2 whose axis passes
through (1, 2, 3) and has direction cosines proportional to (2, − 3, 6).
Solution
2 −3 6
The actual direction cosines of the axis are , , .
7 7 7
∴ The equation of the axis through (1, 2, 3) is
x −1 y − 2 z−3
= = . . . (3)
⎛ ⎞ ⎛
2 − 3 ⎞ ⎛6⎞
⎜ ⎟ ⎜ ⎟ ⎜ ⎟
⎝7⎠ ⎝ 7 ⎠ ⎝7⎠
Let (x, y, z) be any point on the cylinder. Then the perpendicular distance of
(x, y, z) from the axis Eq. (3) will be equal to the radius.
93
Mathematics-I ⎡⎧ 6 ⎛ − 3 ⎞⎫
2
⎧ 2 6⎫
2
⎢ ⎨ ( y − 2) − ( z − 3) ⎜ ⎟ ⎬ + ⎨ ( z − 3) − ( x − 1) ⎬
⎢⎣ ⎩ 7 ⎝ 7 ⎠⎭ ⎩ 7 7⎭

1
2 ⎤2
⎧ ⎛ − 3⎞ 2⎫
+ ⎨( x − 1) ⎜ ⎟ − ( y − 2) ⎬ ⎥ =2
⎩ ⎝ 7 ⎠ 7⎭ ⎥

Squaring,
[(6 ( y − 2) + 3 ( z − 3)]2 + [2 ( z − 3) − 6 ( x − 1)]2 + [3 ( x − 1) + 2 ( y − 2)]2 = 196

Simplifying this, the equation is given by


45 x 2 + 40 y 2 + 13 z 2 + 36 yz − 24 zx + 12 xy − 42 x − 280 y − 126 z + 294 = 0
Example 5.37
Find the equation of the right circular cylinder whose guiding circle is
x 2 + y 2 + z 2 = 9, x − y + z = 3

Solution
The centre of the sphere is (0, 0, 0) and its radius is 3. The perpendicular distance
3
of (0, 0 0) to the given plane is = 3 (in magnitude).
3

Hence the radius of the circle = 9 − 3 = 6 .

P (x, y, z)
M

Figure 5.19

Let P (x, y, z) be any point on the cylinder. The axis passes through (0, 0, 0) and its
direction ratios are (1, − 1, 1), i.e. the direction ratios of the normal to the plane
x − y + z = 3.

The actual direction cosines of the axis are


1 1 1
,− ,
3 3 3
Now OM is the projection of OP on OM and
1 ⎛ 1 ⎞ 1
∴ OM = ( x − 0) + ( y − 0) ⎜ − ⎟ + ( z − 0)
3 ⎝ 3⎠ 3
x− y+z
=
3
94
Three Dimensional
Again OP 2 = x 2 + y 2 + z 2
Geometry

PM 2 = radius 2 = OP 2 − OM 2

or OP 2 = 6 + OM 2

( x − y + z )2
or x2 + y2 + z 2 = 6 +
3

or 3 ( x 2 + y 2 + z 2 ) = 18 + ( x 2 + y 2 + z 2 − 2 xy − 2 yz + 2 zx)

or x 2 + y 2 + z 2 + xy + yz − zx = 9

SAQ 6
(a) Find the equation of the cylinder whose generators are parallel to z-axis and
which passes through the curve of intersection of x 2 + y 2 + z 2 = 1 and
x + y + z = 1.

(b) Find the equation of the cylinder whose generators are parallel to
x y z
= + and which pass through the conic z = 0, 3 x 2 − 4 y 2 = 12 .
2 5 3
(c) Find the equation of the cylinder whose generators are parallel to the line
x y z
= = and whose guiding curve is x 2 + y 2 = 9, z = 1 .
−1 2 3

(d) Find the equation of a cylinder with generators parallel to x-axis which pass
through the curve of intersection of the surfaces represented by
x 2 + y 2 + 2 z 2 = 12, and x − y + z = 1 .

(e) Find the equation of the cylinder whose generators are parallel to the line
y z
x = − = and whose guiding curve is the ellipse x 2 + 2 y 2 = 1, z = 3 .
2 3
(f) Find the equation of a right circular cylinder of radius 2 whose axis passes
through (1, 2, 3) and has direction cosines proportional to (2, − 3, 6).
(g) Find the equation of the right circular cylinder of radius 3 and axis
x −1 y − 3 z − 5
= = .
2 2 −1

(h) Find the equation of the right circular cylinder of radius 2 whose axis is the
x −1 y − 2 z − 3
line = = .
2 1 2

5.7 AREAS AND VOLUMES


In this section we discuss problems on areas and volumes. Such problems arise in the
study of moment of inertia, centre of gravity of plane lamina and solids and various other
fields of practical importance. In the discussion below we first study problems
concerning areas of plane figures and follow them by solving problems involving
volumes of solids.
95
Mathematics-I 5.7.1 Areas Enclosed by Plane Curves
d
Let us consider the definite integral ∫ f ( x) dx . This integral defines the area bounded
c
by the curve y = f (x), the two ordinates x = c and x = d and the x-axis, i.e. the shaded area
in the Figure (5.20).
Y

y = f(x)

A B

X
O
x=c x=d

dx
Figure 5.20

The integral can also be interpreted from the fact that the ordinate y goes from any point
on the x-axis to any point on the curve y = f (x) while x goes in the interval c ≤ x ≤ d .

∴ The integral can also be written as a double integral


d ⎡ f ( x) ⎤ d f ( x)

∫ ⎢ ∫
⎢⎣ 0
dy ⎥ dx =
⎥⎦ ∫ ∫ dy dx ;
c c 0

dy dx is called the element of the area.


In a general way a double integral may be defined as
D f ( x) D ⎡ f ( x) ⎤
∫ ∫ φ ( x, y ) dx dy = ∫ dx ⎢
⎢ ∫ φ ( x, y ) dy ⎥

C g ( x) C ⎣ g ( x) ⎦
Note : To evaluate the double integral, always look to the limits of the inner integral. If
these limits are function of x, integrate w.r.t y first and then w.r.t. x. If on the
other hand, the limits of the inner integral are function of y, integrate w.r.t x first
and then with respect to y.
Example 5.37
Find the area between the curves
y2 = 4x and 2 x − 3 y + 4 = 0

Solution
We first find the points of intersection of the line 2 x − 3 y + 4 = 0 with the
parabola y 2 = 4 x , we have

2x + 4
y=
3

96
Three Dimensional
Substituting for y in the equation y 2 = 4 x , we get Geometry
2
⎛ 2x + 4 ⎞
⎟ = 4 x or x − 5 x + 4 = 0
2

⎝ 3 ⎠
This gives x = 1 and x = 4.
If x = 1, y = 2
and if x = 4, y = 4
∴ The points of intersection are (1, 2), (4, 4). The required area is the shaded area
and is given by
3y − 4
4 2

∫ ∫ 2
dx dy
2 y
4

2x − 3y + 4 = 0
2
y = 4x
B (4, 4)

A (1, 2)

X
O

Figure 5.21

Since the limits are functions of y, we integrate w.r.t. x first and then w.r.t. y.
3y − 4
4 4
⎡ 3y − 4 y2 ⎤
This gives ∫ [ x]
y2
2 dy = ∫ ⎢
⎣⎢ 2
− ⎥ dy
4 ⎥⎦
2 4 2

4
1
4
1 ⎛ 6 y2 y3 ⎞
= ∫ (6 y − 8 − y ) dy = − −
2
⎜⎜ 8 y ⎟
4 4 ⎝ 2 3 ⎟⎠
2 2

1 ⎛ 64 8⎞ 1
= ⎜ 48 − 32 − − 12 + 16 + ⎟=
4 ⎝ 3 3⎠ 3

Note : The above integral is also equivalent to


2 2 x
1
∫ 2 x∫+ 4 dy dx =
3
(same result)
1
3

97
Mathematics-I Example 5.38
Find the area common to the circles
x 2 + y 2 − 4 y = 0, x2 + y2 − 4 x − 4 y + 4 = 0
Y

Q
(0, 2) P
(2, 2)
1 2
X
O A

Figure 5.22
Solution
x 2 + y 2 − 4 y = 0 is a circle with its centre at the point (0, 2) and radius 2, i.e.
circle 1 .
The circle x 2 + y 2 − 4 x − 4 y + 4 = 0 is circle 2 with its centre at the point
(2, 2) and radius also 2. The circle 1 touches the x-axis at the origin while the
circle (2) touches both the axes. The radii of both the circles are 2.
The two circles intersect at the points A and B and their centres at P and Q.
The line AB divides the area of intersection into two equal parts. The area on the
right of this line and the circle 1 is equal to the area on left of this line and the
circle 2 . (By symmetry).
∴ The area between the line AB and the circle 1 is half the area of intersection
between the two circles.
Solving the two equations for the points of intersection, we get
x2 + y2 − 4 x − 4 y + 4 = x2 + y2 − 4 y
This gives x = 1.
Now for the circle x 2 + y 2 − 4 y = 0 , solving for y, we get

4 ± (16 − 4 x 2 )
y= = 2 ± (4 − x 2 )
2
2
2 + (4 − x 2 )
∴ The area A = ∫ ∫2 − (4 − x 2 )
dy dx
1
2
⎡ 2 + (4 = x 2 ) − 2 + (4 − x 2 ) ⎤ dx
= ∫ ⎣⎢ ⎥⎦
1
2 2
⎡x 4 x⎤
=2 ∫ (4 − x ) dx = 2 ⎢ (4 − x ) + sin −1 ⎥
2 2

1
⎣2 2 2 ⎦1

⎡ 3 π⎤ ⎛ 2π 3⎞
= 2 ⎢π − − ⎥ = 2 ⎜⎜ − ⎟
⎣ 2 3⎦ ⎝ 3 2 ⎠⎟

⎛ 2π 3⎞
∴ The total area = 4 ⎜⎜ − ⎟.
⎝ 3 2 ⎟⎠
98
Example 5.39 Three Dimensional
Geometry
Find the area of the region bounded by y 2 = 4 − x and y 2 = x .
y A

P Q

X
O
2
2 y =4-x
y =x

B
Figure 5.23

Solution
To find the area between the two parabolas, we find the points of intersections of
the curves.
Solving the two equations, we get

4 − x = x or x = 2 and y = ± 2

∴ The points are (2, 2) and (2, − 2) .

The area common to the two parabolas is


2 4 − y2 2

∫ ∫ dx dy = ∫ [4 − y 2 − y 2 ] dy
− 2 y2 − 2

2
2
⎛ 2 y3 ⎞ 2
= ∫ (4 − 2 y ) dy = ⎜ 4 y − =8 2 − ×4 2
2
⎜ ⎟
⎝ 3 ⎟⎠ 3
− 2 − 2

16 2
= .
3
Example 5.40
A thin plate of uniform thickness and density equal to ‘1’ covers the region R of
the xy-plane bounded by x 2 = y and x − y + 2 = 0 . Find the moment of inertia Iy
about the y-axis.
Y B (2,4)

A (-1,1)

X
O
Figure 5.24 99
Mathematics-I Solution
Solving the two equations as simultaneous equations, the points of intersection are

x 2 = y = x + 2 or x 2 − x − 2 = 0

∴ x = 2 or x = − 1

If x = 2, y = 4; and if x = − 1, y = 1

The moment of inertia Iy about y-axis is given by


2 x+2 2
Iy = ∫ ∫ x dx dy = ∫ x 2 [ y ]x 2+ 2 dx
2
x
−1 x2 −1

2
= ∫ ( x + 2 − x 2 ) x 2 dx
−1

2
2
⎛ x 4 2 x 3 x5 ⎞
= ∫ ( x + 2 x − x ) dx = ⎜ + −
3 2 4
⎜ 4 ⎟
⎝ 3 5 ⎟⎠
−1 −1

16 32 1 2 1 189 63
=4+ − − + − = =
3 5 4 3 5 60 20
Example 5.41

Find the centroid of gravity of the plane semi-circular region = x 2 + y 2 ≤ 4, x ≥ 0 .

Solution
Since x ≥ 0, and the curve is symmetrical about the x-axis, the centroid lies on the
x-axis, i.e. y = 0.

2 4 − y2 2
x2 (4 − y 2 )
∫ ∫ x dx dy ∫ 2 0
dy
−2 0 −2
∴ x= = 2
4 − y2 (4 − y 2 )

2

∫ ∫
x dy
dx dy 0
−2
−2 0

2
2
4 − y2 1 ⎛ y3 ⎞
∫ 2
dy
2
⎜⎜ 4 y −


3 ⎟⎠
−2 −2
= 2
= 2
⎡ 1 ⎤
∫ 4 − y 2 dy ⎢ y (4 − y 2 ) 2
+ 2 sin −1 y ⎥
−2 ⎢ 2 2 ⎥⎥

⎣ ⎦− 2

8
8
= 3 =
π 3π
Example 5.42

A lamina is bounded by the curves y = x 2 − 3 x and y = 2 x . If the density at any


point is given by λ xy, find the mass of the lamina.
100
Solution Three Dimensional
Geometry
We solve the equations simultaneously to get the points of intersection, i.e.
y = x2 − 3x = 2 x
This gives x = 0 and x = 5. These values of x give the values of y as 0 and 10.

y
(5, 10)

y = 2x

(0, 0) X
2
O y=x –3x

Figure 5.25
The points of intersection are (0, 0); (5, 10).
The mass of the lamina M is given by
5 2x
M = ∫ ∫ 2
λ xy dx dy
0 x − 3x

2x
5
⎡ y2 ⎤ λ
5
=λ ∫ x⎢ ⎥
⎢⎣ 2 ⎥⎦ x 2 − 3 x
dx =
2 ∫ x (4 x 2 − x 4 + 6 x3 − 9 x 2 ) dx
0 0

5
−λ
5
−λ ⎡ x 6 6 x5 5 x 4 ⎤
= ∫ ( x − 6 x + 5 x ) dx = ⎢ − +
5 4 3

2
0
2 ⎣⎢ 6 5 4 ⎥⎦
0

4375
= λ
12
5.7.2 Volumes of Solids
Let us first define Volume Integrals or Triple Integrals. Consider the function
f (x, y, z) defined at every point of the region V in space. Then the triple integral of the
function over the region V is defined by

∫∫∫ f ( x, y, z ) dV

For purposes of evaluation, this integral is usually expressed as an iterated integral and is
written as

∫∫∫ f ( x, y, z ) dx dy dz

More specifically if the order of the differentials is dz dy dx, then we should have the
related integrals written as
x2 ⎧ y2 ⎡ z2 ⎤ ⎫
⎪ ⎢ f ( x, y, z ) dz ⎥ dy ⎪⎬ dx
∫ ∫ ∫⎨
⎪⎩ y1 ⎢ ⎥ ⎪
x1 ⎣ z1 ⎦ ⎭
where z1, z2 may be constant or functions of x and y; y1, y2 may be constant or functions
of x; and x1, x2 in the last integral in that order are constants.
101
Mathematics-I Example 5.43
1 z x+ z
Evaluate ∫ ∫ ∫ ( x + y + z ) dx dy dz
−1 0 x − z

Solution
The given integral can be written as
1 ⎧z ⎡x+ z ⎤ ⎫
⎪ ⎪
∫ ∫ ∫ ⎨ ⎢

( x + y + z ) dy ⎥ dx ⎬ dz
⎥ ⎪
−1 ⎩⎪ 0 ⎣x− z ⎦ ⎭

1 ⎡z ⎡ 2 ⎤x+ z ⎤
⎢ ⎢( x + z ) y + y ⎥
= ∫ ∫ ⎢ ⎣⎢ 2 ⎥⎦
dx ⎥ dz

−1 ⎣0 x−z ⎦
1 ⎡z ⎡ ( x + z )2 ( x − z )2 ⎤ ⎤
= ∫ ∫ ⎢ + + − − − ⎥ dx ⎥ dz
2 2 2
⎢ ( x z ) ( x z )
−1 ⎢⎣ 0 ⎢⎣ 2 2 ⎥⎦ ⎥⎦

1 ⎡z ⎤
=2 ∫ ∫ ⎢ ( z 2 + 2 x z ) dx ⎥ dz
⎢⎣ 0 ⎥⎦
−1

z
1
=2 ∫ z 2 x + x2 z dz
−1 0

1
1
z4
=2 ∫ ( z + z ) dz = 4 =0
3 3
4
−1 −1

5.7.3 Volume as Double Integrals


Consider the surface z = f ( x, y ) and let its orthogonal projection on the xy-plane be the
area S. Then the volume of the solid cylinder on S as base bounded by the given surface
having generators parallel to the axis of z is given by

∫∫ z dx dy = ∫∫ f ( x, y ) dx dy

where the integration is to be performed over the area S.


Suppose again it is required to find the volume of the solid bounded above and below by
surfaces z = f ( x, y ) and z = g ( x, y ) respectively and laterally by a cylinder with axis
parallel to z-axis, then the volume of the solid is given by
f ( x, y )

∫∫ ∫ dz dy dx
R g ( x, y )

where R denotes the region of the xy-plane denoted by the cylinder and the orthogonal
projection of the solid in the xy-plane.
Example 5.44
Find the volume of the solid enclosed by the surfaces

x 2 + y 2 = cz , x 2 + y 2 = 2ax, z = 0 .
102
Solution Three Dimensional
Geometry
The volume of the solid is given by

V = ∫∫ z dx dy
R

x2 + y 2
where the surface z = is the parabolid of revolution and the region R is
c
the circle in the xy-plane. The limits for y are − (2ax − x 2 ) to (2ax − x 2 ) and
those for x are 0 to 2a.

2a (2 ax − x 2 )
x2 + y 2
∴ V = ∫ ∫ c
dx dy
0 − (2 ax − x 2 )

2a (2 ax − x 2 )
2 y3
=
c ∫ x2 y +
3
dx
0 0

2a
2
=
3c ∫ (2ax − x 2 ) [3x 2 + 2ax − x 2 ] dx
0

2a
2
=
3c ∫ x (2a − x) (2 x 2 + 2ax) dx
0

Put x = 2a sin 2 θ, dx = 4a sin θ cos θ dθ

when x = 0, θ = 0,

π
when x = 2a, θ=
2
∴ The given integral can be transformed as
π
2
2
3c ∫ 2a sin θ 2a cos θ (8a 2 sin 4 θ + 4a 2 sin 2 θ) 4a sin θ cos θ dθ
0

π
4 2
64a
=
3c ∫ (2 sin 6 θ cos 2 θ + sin 4 θ cos 2 θ) d θ
0

64a 4 ⎡ 2 . 5 . 3 .1.1. π 3 .1. π ⎤


= ⎢ + ⎥
3c ⎣2 . 4 . 3 . 2 . 2 . 2 . 2 . 2 2 . 3 . 2 . 2 . 2 . 2⎦

a 4 ⎛ 5π ⎞ 9π a
4
3π a 4
= ⎜ + 2 π ⎟ = =
3c ⎝ 2 ⎠ 6 2

π m +1 n +1
2
2 2
Note : ∫ sin m θ cos n θ d θ = (Gamma function).
m+n+2
0
2
2
103
Mathematics-I Example 5.45

Find the volume bounded by the cylinder x 2 + y 2 = 4 and the planes


y + z = 4, z = 0 .

Solution
Using double integral, the volume is given by

2 (4 + x 2 )
V = ∫ ∫ (4 − y ) dx dy
− 2 − 4 − x2 )

2 2 4 − x2 )
y
= ∫ 4y −
2 2
dx
−2 − 4− x )

2 2
= ∫ 8 (4 − x 2 ) dx = 16 ∫ (4 − x 2 ) dx
−2 0

Put x = 2 sin θ, dx = 2 cos θ d θ


π π
2 2
∴ V = 16 ∫ 4 cos 2 θ d θ = 32 ∫ (1 + cos 2θ) d θ
0 0

π
sin 2θ 2
= 32 θ + = 16π
2 0

5.7.4 Volume as Triple Integral


A triple integral is defined much in the same way as the double integral. The given solid
is divided by planes parallel to the co-ordinate planes into rectangular parallelopipeds of
elementary volume δx δy δz . The total volume is therefore, represented by

∫ ∫∫ dx dy dz

with suitable limits of integration. The quantity ‘dx dy dz’ is called the element of the
volume.
Example 5.46
log 2 x x + y
Evaluate ∫ ∫ ∫ e x + y + z dx dy dz .
0 0 0

Solution
log 2 x x+ y log 2 x

∫ ∫ ∫ ∫
x+ y+z
= [e ] dy dx = (e 2 x + 2 y − e x + y ) dy dx
0 0 0 0 0

x
⎡ e2 x + 2 y
log 2 log 2
⎤ ⎛ e4 x e2 x ⎞
= ∫ ⎢
⎢⎣ 2
− e x + y ⎥ dx =
⎥⎦ 0
∫ ⎜⎜
⎝ 2

2
− e2 x + e x ⎟ dx


0 0

⎛ e4 x 3 2 x ⎞
log 2
= ∫ ⎜⎜
⎝ 2
− e + e x ⎟ dx
2 ⎟

0
104
log x Three Dimensional
⎡ e4 x 3 2 x ⎤ Geometry
=⎢ − e + ex ⎥
⎢⎣ 8 4 ⎥⎦ 0

16 3 ⎛1 3 ⎞
= − × 4 + 2 − ⎜ − + 1⎟
8 4 ⎝8 4 ⎠

3 5
=2−3+2− =
8 8

5.7.5 Change of Variables


In the solution of problems in multiple integrals an appropriate choice of system of
co-ordinates plays a very important role as it very greatly facilitates the evaluation of
such integrals. Before proceeding to solve the problems, we discuss three co-ordinate
systems, study their relationship and use them in the evaluation of areas and volumes.
Note : A brief discussion of two special curvilinear systems of co-ordinates.

5.7.6 Cylindrical Co-ordinates (ρ, φ, z)


Any point P (x, y, z) whose projection on the xy-plane is the point Q (x, y) has its
cylindrical co-ordinates denoted by (ρ, φ, z), where
ρ = OQ, φ = ∠ XOQ and z = QP
Z

dφ ρ dφ

dz
P (ρ, φ, z)

O
Y

x
φ ρ
y
Q (ρ, φ)

X
Figure 5.26

The level surfaces represented by ρ = ρ0 , φ = φ0 and z = z0 are respectively cylinders


about z-axis; planes through the z-axis; and planes perpendicular to z-axis.
The co-ordinate curves for ρ are the rays perpendicular to the z-axis; for φ, horizontal
circles with their centers on the z-axis; and for z, lines parallel to the z-axis.
∴ From the figure, we have
x = ρ cos φ; y = ρ sin φ and z = z.
This gives the relationship between the Cartesian co-ordinates and the cylindrical
co-ordinates.
(a) The element of the volume dv is given by
dv = dx dy dz = (d ρ) (ρd φ) dz
105
Mathematics-I Z
P (ρ, φ, z)

O
Y
ρ
φ
N

Figure 5.27

(b) The element of the area dA is given by


dA = dx dy = (d ρ) (ρd φ) .

V = ∫ ∫ ∫ ρ d ρ d φ dz
A= ∫ ∫ ρ dρ dφ
5.7.7 Spherical Polar Co-ordinates (r, θ, φ)
Let P (x, y, z) be any point in space whose projection on the xy-plane is the point
Q (x, y). Then the spherical polar co-ordinates of the point are (r, θ, φ), where r = OP;
θ = ∠ ZOP and φ = ∠ XOQ.

r sin θ d φ

M
r sin θ

N
dr
P

θ r H
L
r dθ

O z
y

φ dφ
x r sin θ

Y Q
X

Figure 5.28

The level surfaces r = r0; θ = θ0; φ = φ0 are respectively spheres about O; cones about the
z-axis with vertex at the origin; and planes through the z-axis.

106
Three Dimensional
Z Geometry
P (x, y, z)
(r, θ, φ)

r
θ

z
y
O

x φ

y Q

x
Figure 5.29

The co-ordinate curves for r are rays from the origin; for θ, vertical circles with centre at
O (called meridian); for φ, horizontal circles with centers on the z-axis.
∴ From the figures, we get
x = OQ cos φ = OP sin θ cos φ = r sin θ cos φ

y = OQ sin φ = OP sin θ sin φ = r sin θ sin φ

z = OQ cos θ = r cos θ

(a) The element of the volume dv is


dv = ( PN ) ( PL) ( PM ) = dr . r d θ . r sin θ d φ

= r 2 sin θ dr d θ dφ

(b) The element of the area dA is


dA = dr × r d θ = r dr d θ

Particular Cases
(a) To change Cartesian co-ordinates (x, y) to polar co-ordinates (r, θ), we have
the relation x = r cos θ, y = r sin θ and then

∫ ∫ f ( x, y ) dx dy = ∫ ∫ f ( r cos θ, r sin θ) r dr d θ
Rx y Rr θ

(b) To change rectangular co-ordinates (x, y, z) to cylindrical co-ordinates


(ρ, φ, z), we have the relation x = ρ cos φ, y = ρ sin φ, z = z , and then

∫∫ ∫ f ( x, y , z ) dx dy dz = ∫∫ ∫ f (ρ cos φ, ρ sin φ, z ) ρ d ρ d φ dz
Rxyz Rρθ z

(c) To change rectangular co-ordinates (x, y, z) to spherical polar co-ordinates


(r, θ, φ), we have the relation x = r sin θ cos φ, y = r sin θ sin φ, z = r cos θ
and then

∫∫ ∫ f ( x, y, z ) dx dy dz = ∫∫∫ f ( r sin θ cos φ, r sin θ sin φ, r cos θ)


Rxyz

r 3 sin θ dr d θ d φ

Example 5.47 107


Mathematics-I a
(5ax − x 2 ) ( x2 + y 2 )
Evaluate ∫ ∫2 ax y2
dx dy .
0

Solution
The region of integration is the shaded portion of the area in Figure 5.30.
Y

(a, 2a)

C
X
O (5a/2, 0)

Figure 5.30

The point of intersection of the parabola y 2 = 4ax , the circle x 2 + y 2 − 5ax = 0


and the line x = a is the common point A. To solve the problem, we transform the
integral from Cartesian system to polar co-ordinate, i.e.
x = r cos θ, y = r sin θ, dx dy = r d θ dr

The limits correspondingly are changed to

y 2 = 4 ax

∴ r 2 sin 2 θ = 4ar cos θ

4a cos θ
∴ r=
sin 2 θ

or y 2 = (5ax − x 2 )

or x 2 + y 2 = 5ax

or r 2 = 5ar cos θ; r = 5a cos θ

4a cos θ
∴ The radial strip has its lower end on the parabola r = and its upper
sin 2 θ
end on the circle r = 5a cos θ .

To determine the limits for θ, we notice that the radius vector rotates from the line
π
OA where θ = tan −1 2 to the line θ = .
2
∴ The transformed integral

108
π Three Dimensional
2 5a cos θ Geometry
r
I= ∫ ∫
4 a cos θ r sin 2 θ
2
r dr d θ
tan −1 2
sin 2 θ

π π
2
dθ 2 ⎛ 4a cos θ ⎞ dθ
∫ ∫
θ
= [r ]54aacos = ⎜⎜ 5a cos θ − ⎟
sin 2 θ ⎟⎠ sin 2 θ
cos θ
sin 2 θ ⎝
tan −1 2 sin 2 θ tan −1 2

π π
2 2
= ∫ 5a cosec θ cot θ d θ + 4a ∫ cosec 2 θ (− cosec θ cot θ) d θ
tan −1 2 tan −1 2

π
π
⎡ cosec3 θ ⎤ 2
= 5a [− cosec θ] 2 −1 + 4a ⎢ ⎥
⎣⎢ 3 ⎦⎥ tan −1 2
tan 2

5a 4a 4a 5 5
= − 5a + 5+ − .
2 3 3 8
11a 5 5 a ⎛ 1⎞ a
=− + ⎜1 − ⎟ = (5 5 − 11) .
3 2 ⎝ 3⎠ 3
Example 5.48
Evaluate the double integral
a
(a2 − x2 ) xy
∫ ∫ (ax − x )
2
+ y2 )
e− ( x dx dy
x +y
2 2 2
0

Solution
The region of integration is the shaded portion of the area in Figure 5.31.
Y
B (0, b)

θ
X′ X
O (0, 0)
A (a, 0)
(a/2, 0)

Y′

Figure 5.31

The lower limit of the inner integral is the circle x 2 + y 2 − ax = 0 , while the
upper limit is x 2 + y 2 = a 2 . The centre and the radius of the inner circle are
⎛a ⎞ a
⎜ , 0 ⎟ and and the centre and radius respectively of the outer circle are (0, 0)
⎝2 ⎠ 2
and a, while for the outer integral x = 0 to x = a.
As done in the earlier example, we transform the given integral from Cartesian
co-ordinates to polar co-ordinates. The transformed integral is written as 109
Mathematics-I π
2 r =a r 2 sin θ cos θ
∫ ∫r = a cos θ
2
e − r r dr d θ
0
r2
π
2
1 a
∫ ∫a cos θ
2
=− sin θ cos θ (− 2r e − r ) dr d θ
2
0

π
2
1

2
=− [e − r ]aa cos θ sin θ cos θ d θ
2
0

π
2
1

2 2
cos 2 θ
=− (e − a − e − a ) sin θ cos θ d θ
2
0

π π

1 2
sin 2θ 1 2

∫ ∫
− a2 2
cos 2 θ
=− e dθ + e− a sin θ cos θ d θ
2 2 2
0 0

π
2
1 − a2 1 1

2
cos 2 θ
=− e + . 2 e− a (2a 2 sin θ cos θ) d θ
4 2 2a
0

π
1 − a2 1 2 2 1 2 1 2
=− e + 2 [e− a cos θ ]02 = − e− a + 2 (1 − e− a )
4 4a 4 4a
Example 5.49
1 2 1
Find the volume bounded by the surfaces z = 4 − x 2 − y and z = 3x 2 + y 2 .
4 4
z

1 2
z = 3x 2 + y
4

1 2
z = 4 − x2 − y
4

X
Figure 5.32

Solution
The intersection of the two parabolids is given by
1 2 1
4 − x2 − y = 3x 2 + y 2
4 4
1 2
or 4x2 + y =4
2
110
Three Dimensional
i.e. 8x2 + y 2 = 8 Geometry
The required volume is given by
1 2
4 − x2 − y
V = ∫ ∫ ∫3x 2
+
4
1 2
y
dx dy dz
Rxy 4

where Rxy is the ellipse 8 x 2 + y 2 = 8


1
8 − 8 x2 ⎛ 1 2 1 2⎞
V =4 ∫ ∫0 ⎜ 4 − x − y − 3x − y ⎟ dx dy
2 2

0
⎝ 4 4 ⎠

1 (8 − 8 x 2 )
⎡ 1 3⎤
V =4 ∫ ⎢ 4 (1 − x ) y − 6 y ⎥
2
dx
0
⎣ ⎦0

1 ⎡ 3
8 2
3⎤
=4 ∫ ⎢8 2 (1 − x ) − (1 − x ) ⎥ dx
2 2 2 2

0
⎢⎣ 3 ⎥⎦

1 3
64 2
= ∫ (1 − 2 2
x ) dx
3
0

Put x = sin θ
dx = cos θ d θ
π π
2 2
64 2 64 2
∴ V =
3 ∫ cos3 θ cos θ d θ =
3 ∫ cos 4 θ d θ
0 0

64 2 3π
= × = 4 2π
3 16
Example 5.50
Find the volume of the region bounded above by the sphere r = 2a and below by
θ = α (0 < α < π) using spherical polar co-ordinates.
Z

θ=α

θ
α r = 2a

o Y

X
Figure 5.33

Solution
In the limit, r varies from r = 0 to r = 2a; θ varies from θ = 0 to θ = α and φ varies
from φ = 0 to φ = π/2. 111
Mathematics-I The volume V in terms of spherical polar co-ordinates is given by
π
2 α 2a
V =4 ∫∫ ∫ r 2 sin θ dr d θ d φ (By symmetry)
0 0 0

π
2 α 2a
=4 ∫ dφ ∫ sin θ d θ ∫ r 2 dr (By symmetry)
0 0 0

⎛π⎞ ⎛ (2a )3 ⎞
= 4 ⎜ ⎟ (1 − cos α ) ⎜
⎝2⎠ ⎜ 3 ⎟⎟
⎝ ⎠
16π a3
= (1 − cos α)
3
SAQ 7
a ( a2 − x2 )
⎡π ⎤
(a) Evaluate ∫ ∫ sin ⎢ 2 ( a 2 − x 2 − y 2 ) ⎥ dx dy .
⎣a ⎦
0 0
1
y2 +1
(b) Evaluate ∫ ∫y x 2 y dx dy .
0
2a
(2 a 2 − x 2 )
(c) Evaluate ∫ ∫0 ( x 2 + y 2 ) dx dy .
0
∞ ∞ ∞
π
∫ ∫ ∫
2
+ y2 ) 2
(d) Evaluate e− ( x dx dy . Hence show that e− x dx = .
2
0 0 0

(e) Sketch the area of double integration and evaluate


a
2
(a2 − y2 )
∫ ∫y log e ( x 2 + y 2 ) dx dy
0

( x2 + y 2 )2
(f) Evaluate the double integral ∫∫ x2 y 2
dx dy over the area common

to the circle
x 2 + y 2 = ax ; x 2 + y 2 = by (a > b > 0)

(g) Find the area between the parabola y = x 2 − 6 x + 3 and the line
y = 2x − 9 .

(h) Find the volume cut off from the sphere x 2 + y 2 + z 2 = a 2 by

(i) the cone x 2 + y 2 = z 2

(ii) the cylinder x 2 + y 2 = ax


[Hint : Use cylindrical co-ordinates].

112
a Three Dimensional
(i) A right circular cylinder of radius and height a is formed by the planes Geometry
2
z = 0, z = a and the surface x 2 + y 2 = ax . Find the volume of the portion of
the cylinder inside the cone x 2 + y 2 = z 2 .
(j) Find by double integration the area included between the curves

y 2 = 4a ( x + a ) and y 2 = 4b (b − x )

(k) Find the volume bounded by the parabolid x 2 + y 2 = az , the cylinder


x 2 + y 2 = 2ay and the plane z = 0.

(l) Find the centroid of the region bounded by the paraboloid z = 4 − x 2 − y 2


and xy-plane. (Assuming constant density ρ).
[Hint : CG lies on z-axis. The CG formula is

z =
∫ ∫ ∫ x dx dy dz , x = y = 0 .]
∫ ∫ ∫ dx dy dz

5.8 NUMERICAL INTEGRATION AND SIMPSON’S


RULE
b
In the earlier section we have shown that the ∫ f ( x ) dx represents the area between the
a
curve y = f ( x) , the two co-ordinates x = a and x = b and the x-axis. The integral can be
computed in closed form when the function f (x) is integrable in finite terms. There are
however, situations when the integrand function f (x) cannot be integrated exactly or the
direct integration becomes too cumbersome or unwieldy to perform. In such situations
b
the integral ∫ f ( x) dx is evaluated by numerical methods.
a

In dealing with the computation of the problems by numerical methods, the function f (x)
is given by a set of numerical values for the corresponding values of the independent
variable x. Numerical integration is then performed to get at the close value of the
integral. A number of approximate methods have been suggested to this end. Among the
most commonly and widely used methods for numerical integration are the ‘Trapezoidal
Rule’ and the ‘Simpson’s Rule’ which will be derived and discussed in the succeeding
sections.

5.8.1 The Trapezoidal Rule


Let AB be the curve y = f (x) and let AM and BN be the ordinates y1, and yn + 1 respectively
drawn at the terminal points A and B of the curve.
Let OM = a and ON = b so that MN = b – a. Divide the line segment MN into ‘n’ equal
b−a
parts, i.e. MM1, M1M2 . . . Mn – 1 B so that each part = = h (say).
n
Draw ordinates at the points M1 , M 2 , . . . M n −1 and let them be denoted by
y1 , y2 , . . . yn −1 respectively. Join the ends of the consecutive ordinates on the curve by
straight lines. This gives us n trapezium each of which will have the same width.
113
Mathematics-I Let S1 , S 2 , . . . S n be the respective areas of these trapeziums.

Y
B

y = f (x)
A
yn + 1
y1 y2 h
y3
a
h h
X
O M M1 M2 Mn N

Figure 5.34

1
Then S1 = ( y1 + y2 ) h
2
1
S2 = ( y2 + y3 ) h
2
................
................
................
1
Sn = ( y n + yn + 1 ) h
2
Adding all these elementary areas, the total area under the curve will be given by
1
S1 + S2 + . . . + Sn = ( y1 + y2 + y2 + y3 + . . . + yn + yn +1 ) h
2
1
= h [( y1 + yn +1 + 2 ( y2 + y3 + . . . + yn )]
2
1
= h [ P + 2Q]
2
where P = sum of the first and the last ordinate, and
Q = sum of the remaining ordinates.
b
But the whole area MABN = ∫ y dx
a
b
1
∴ ∫ y dx =
2
h ( P + 2Q) . . . (1)
a

Relation (1) is known as Trapezoidal Rule. Geometrically this rule signifies that the
curve y = f (x) is approximated by n straight lines joining the points
[(a, y1 ), (a + h, y2 )]; [(a + h, y2 );(a + 2h, y3 )] ; . . . ; . . . ; [(a + (n −1) h, yn ),(a + n h, yn + 1 )] .

It also establishes the fact that the area bounded by the curve y = f (x), the two ordinates x
= a and x = b and the x-axis is approximately equal to the sum of n trapeziums.
We can remember this formula as
⎛h⎞
⎜ ⎟ [First ordinate + last ordinate + 2 (sum of the remaining ordinates)].
⎝2⎠
114
5.8.2 Simpson’s Rule Three Dimensional
Geometry
A better approximation to the area AA′P′P can be obtained by using a better method
known as Simpson’s rule.
Let A′ B ′ C ′ D ′ . . . P ′ be any curve defined by y = f (x) and let equidistant ordinates
y1 , y2 , y3 , y4 , y5 , . . . be drawn at equal intervals of width h meeting the curve at the
points A′ B ′ C ′ D ′ E ′ . . . .

Since the area is to be computed in terms of the ordinates, we can fix the origin at any
point on the x-axis,

P′

D′ E′
C′
B′
A′

y1 y2 y3 y4 y5

h h h h h

O A B C D E P

Figure 5.35

Let us choose the origin at the point B so that the x-co-ordinates of the points A and C are
– h and h respectively.
To calculate the area under the curve, we fix three points A′ B ′ C ′ and pass a parabola
through these points with its axis vertical. The equation of the curve is represented by

y = a + bx + cx 2

The area of the double strip AA′ C ′ C is given by


h
h h
⎡ bx 2 cx3 ⎤
∫ y dx = ∫ (a + bx + cx ) dx = ⎢ ax + +
2

−h −h ⎣⎢ 2 3 ⎥⎦
−h

2h
= (3a + ch2 )
3
The co-ordinates of the points A′, B ′, C ′ are A′ ( − h, y1 ); B ′ (0, y2 ); C ′ ( h, y3 ) . As the
curve passes through these points, this gives

y1 = a − bh + ch 2 . . . (2)
y2 = a . . . (3)

y3 = a + bh + ch 2 . . . (4)
From (2) and (4)

y1 + y3 = 2 ( a + ch 2 ) = 2 ( y2 + ch 2 )

y1 + y3 − 2 y2
∴ c= ; 2bh = y3 − y1
2h 2
115
Mathematics-I y3 − y1
∴ b=
2h
Substituting for a, b, c the area AA′ C ′ C is given by

2h ⎡ y1 + y3 − 2 y2 ⎤ 2h ⎛ y1 + 4 y2 + y3 ⎞
⎢3 y2 + ⎥= 3 ⎜ ⎟
3 ⎣ 2 ⎦ ⎝ 2 ⎠
h
= ( y1 + 4 y2 + y3 ) . . . (5)
3
Relation (5) gives us the area of the double strip AA′ C ′ C .
h
Similarly, the area of the next double strip CC ′ E ′E is ( y3 + 4 y4 + y5 ) and so on.
3
Now let the number of such double strips into which the entire area is divided by n so
that the total number of ordinates are (2n + 1).
The total area under the curve between the ordinates y1 and y2n + 1 will be the sum of all
such elementary areas and is given by
h
[( y1 + 4 y2 + y3 ) + ( y3 + 4 y2 + y5 ) + . . . + ( y2 n −1 + 4 y2 n + y2 n +1 )]
3
h
= [( yn + y2 n +1 ) + 2 ( y3 + y5 + . . . + y2n −1 ) + 4 ( y2 + y4 + . . . + y2 n )] . . . (6)
3
Relation (6) is known as Simpson’s 1/3-Rule.
It is important to note that while using this rule, the interval of integration must be
divided into even number of sub-intervals each of width ‘h’, so that there are odd
numbers of ordinates. This rule is a closed formula for the simple reason that the end
ordinates are also included in the formula.
We can remember this formula as
⎛h⎞
⎜ ⎟ [(First Ordinate + Last Ordinate) + 2 (sum of odd ordinate)
⎝3⎠
+ 4 (sum of even ordinates)]
5.8.3 Simpson’s 3/8-Rule
This rule we state without proof. In this rule, the elementary block is taken to consist of
three sub-divisions or four ordinates. Thus, polynomial of degree three is taken for
purposes of calculation of elementary area and the procedure is continued in this manner
to cover the entire area.
a + 3h 3h
∴ ∫a f ( x) dx =
8
( y1 + 3 y2 + 3 y3 + y4 )

a + 6h 3h
Similarly, ∫a + 3h f ( x) dx =
8
( y4 + 3 y5 + 3 y6 + y7 )

The sum of all such integrals will give the total area.
This is known as Simpson’s 3/8 Rule.
It must be born in mind that while applying this rule the number of sub-divisions should
be taken as multiple of three.
5.8.4 Applications of Simpson’s Rule
It is important to note that
(a) If the ordinates represent cross-sectional areas at equal intervals, then this
rule provides the volume of the solid.
116
(b) If the y-values or the ordinates denote velocities at equal interval of times, Three Dimensional
then the Simpson’s rule will give the distance travelled. Geometry

Thus, the method can be used to determine approximately from the observed data either
an area, or volume or the work done, etc.
Example 5.51
The velocity of a train which starts from rest is given by the following table, the
time being reckoned in minutes from the start and speed in km per hour.
Minutes 2 4 6 8 10 12 14 16 18 20
km per 10 18 25 29 32 20 11 5 2 0
Hour

Estimate approximately the total distance run in 20 minutes.


Solution

We know that ∫ v dt = distance travelled. Hence to get the distance, we may


regard different speeds as ordinates. The number of ordinates being even, we can
introduce one more ordinate as at t = 0. Speed per hour is zero, since the train
starts form rest.
2 1
Here h = = ; since the speed is in km/h.
60 30
X = 0 + 0 = 0 = sum of the first and the last ordinate
O = sum of odd-ordinates = 18 + 29 + 20 + 5 = 72
E = sum of even-ordinates = 10 + 25 + 32 + 11 + 2 = 80
∴ The required distance
1 464
= (0 + 144 + 320) = = 5.16
3 × 30 90
Example 5.52
A tank is discharging water through an orifice at a depth x m. below the surface
whose area is A sq. m. The following are the values of x for the corresponding
values of A
A 12.57 13.9 15.2 16.5 18.09 19.62 21.23 22.95 24.62 26.5 28.27
x 5 5.5 6 6.5 7 7.5 8 8.5 9 9.5 10

10
A
Using the formula (0.301) T = ∫ x
dx to calculate T, the time in seconds, for
5
the level of water to drop from 10 m to 5 m above the orifice.
Solution
A 12.5
Here y = ; y1 = = 5.620
x 5
y2 = 5.913 y3 = 6.204 y4 = 6.471

y5 = 6.837 y6 = 7.161 y7 = 7.523

y8 = 7.872 y9 = 8.208 y10 = 8.598

1
y11 = 8.939 and h =
2
∴ By Simpson’s Rule 117
Mathematics-I 10
A 1 1
∫ x
dx = . [5.620 + 8.939 + 4 (5.913 + 6.471 + 7.161 + 7.872 + 8.598)
3 2
5

+ 2 (6.204 + 6.837 + 7.523 + 8.208)]

1
= (216.163) = 36.02 T
6
∴ T (0.301) = 36.027
∴ T = 119.7 seconds .
Example 5.53
A body is in the form of a solid of revolution. The diameter D in cms of its
sections at distances x. cm. from one end are given below. Estimate the volume of
the solid
x 0 2.5 5.0 7.5 10.0 12.5 15.0
D 5 5.5 6.0 6.75 6.25 5.5 4.0

Solution
Here h = 2.5. Since the body is rotating, the volume of the solid generating is given
by
15 15
D2 π 2.5
∫ π
4
dx = .
4 3 ∫ D 2 dx
0 0


= [(25 + 16) + 4 (30.25 + 45.5625 + 30.25) + 2 (36 + 39.0625)]
24

= [41 + 4 × 106.0625 + 2 × 75.0625]
24

= (41 + 424.25 + 150.125) = 402.5578
24
Example 5.54
A rocket is launched from the ground. Its acceleration is registered during the first
80 seconds and is given in the table below. Using Simpson’s 1/3-rule, find the
velocity of the rocket at t = 80 seconds.

t (sec) 0 10 20 30 40 50 60 70 80
2
f (cm/sec ) 30.0 31.63 33.34 35.49 37.75 40.33 43.25 46.69 50.67

Solution
Here h = 10; using Simpson’s 1/3-rule, the velocity of the rocket is given by
10
[(30.0 + 50.67) + 4(31.63 + 35.49 + 40.33 + 46.69) + 2(33.34 + 37.75 + 43.25)]
3
10 10
= [(80.67 + 4 × 154.12 + 2 × 114.34) = (80.67 + 616.48 + 228.68)
3 3
10
= × 925.83 = 3086.1 cm/sec.
3
Example 5.55
118
A curve is drawn to pass through the points given by the following table : Three Dimensional
Geometry
x 1.0 1.5 2.0 2.5 3.0 3.5 4.0
y 2 2.4 2.7 2.8 3 2.6 2.1

Y
L

M P (x, y)

dx

X
O A Q B

Figure 5.36
Find
(a) The area bounded by the curve, the x-axis and the lines x = 1 and
x = 4.
(b) The CG of the area.
(c) The volume of revolution of the area about OX.
(d) The radius of gyration of the area about OX.
Solution
(a) Let the curve be given by ML.
OA = 1 and OB = 4.
4
The area ABLM = ∫ y dx
1

0.5
= [2 + 2.1 + 4 (7.8) + 2 (5.7)] = 7.87 sq. units.
3
4
(b) The volume of revolution about OX is π ∫ y 2 dx
1

0.5
= 3.14 [8.41 + 4 × 20.36 + 2 × 16.29] ×
3
122.43
= 3.14 × = 3.14 × 20.41 = 64.07 cubic units.
6
Let P (x, y) be any point on the curve LM. PQ is an elementary strip of the
area of width dx and of length PQ = y.
Elementary mass of the strip = y dx . ρ.
where ρ is the density of the unit area.
4
The total mass = ∫ ρ y dx
1

⎛ y⎞
The co-ordinates of the CG are located at the mid-point ⎜ x, ⎟ of the strip.
⎝ 2⎠
If x , y are the co-ordinates of the CG of the area, then
119
Mathematics-I 4 4
1
∫ ρ xy dx
2 ∫ ρ y 2 dx
x= 1
4
; y= 4
1

∫ ρ y dx ∫ ρ y dx
1 1

PQ can be considered to be a uniform rod of length y. Hence its moment of


inertia about OX is given by
1 1
= × Mass of the rod × PQ 2 = ρy dx . y 2
3 3
4
ρ
∴ Moment of inertia of the whole area =
3 ∫ y 3 dx .
1

If k is taken as the radius of gyration, then


4
ρ
Moment of inertia 3 ∫ y 3 dx
k2 = = 1
4
Mass
ρ ∫ y dx
1

To integrate the various integrals that have appeared in various formulae


above, we prepare the following table to facilitate the computational work.
x y xy y2 y3
1.0 2 2 4 8
1.5 2.4 3.6 5.76 13.824
2.0 2.7 5.4 7.29 19.683
2.5 2.8 7.0 7.84 21.952
3.0 3.0 9.0 9.00 27.000
3.5 2.6 9.1 6.76 17.576
4.0 2.1 8.4 4.41 9.261

(c) CG of the area is calculated as follows


4
0.5
∫ xy dx =
3
[10.4 + 4 × 19.7 + 2 × 14.4]
1

118.0
= = 19.67 approximately.
6
For the integral
4
0.5
∫ y 2 dx =
3
[8.41 + 4 × 20.36 + 2 × 16.29]
1

122.43
= = 20.41 approximately.
6
Hence substituting the approximate values
19.67
x= (From (a)) = 2.53
7.78
1 20.41
y= = 1.31
2 7.78
120
(d) Radius of gyration Three Dimensional
Geometry
4
0.5
∫ y 3 dx =
3
[8.0 + 9.261 + 4 (17.576) + 2 (46.683)]
1

1
= (180.931) = 30.16
6
4
1
3 ∫ y 3 dx
1 30.16
Hence, k2 = 4
1
= × = 1.29 .
3 7.78
∫ y dx
1

SAQ 8
(a) Compute the value of the integral
1.4

∫ (e x + sin x − log x ) dx
0.2

by (i) Trapezoidal rule


(ii) Simpson’s 1/3-rule
Divide the interval of integration into 6 parts.
1
dx
(b) Evaluate ∫ 1 + x2
, using
0

1
(i) Simpson’s 1/3-rule, take h =
4
1
(ii) Simpson’s 3/8-rule, take h =
6
Hence, compute the approximate value of π in each case.
(c) A solid of revolution is formed by rotating about x-axis the area enclosed by
the ordinates x = 0 and x = 1 and a curve through the points given by
(0, 1.0), (0.25, 0.9896), (0.50, 0.9589), (0.75, 0.9089), (1.0, 0.8415).
Calculate the volume of the solid formed using of Simpson’s 1/3-rule.
(d) A mound of earth 3 ft high in the centre is in the form of a solid of
revolution with a vertical axis. The radius of section of the mound at various
heights is given in the following table. Estimate the volume of the mound
using Simpson’s 1/3-rule.
Height (in ft) 0 0.5 1.0 1.5 2.0 2.5 3.0
Radius (in ft) 6 4.5 3 2 1 0.5 0

(e) A curve is drawn to pass through the points which are given in the following
table

x 0 1 2 3 4 5 6
y 0 2 2.5 2.3 2 1.7 1.5

Find
(i) The area bounded by the curve, the x-axis and the lines x = 0
and x = 6.
(ii) The centre of gravity of the area. 121
Mathematics-I (iii) The volume of revolution of the area about OX.
(iv) The radius of gyration of the area about OX.
(f) A reservoir discharging water through sluices at a depth h below the water
surface has a surface area A for various values of h as given by the following
table.

h 10 11 12 13 14
A 950 1070 1200 1350 1530

dh h
If t denotes the time, the rate of fall of the surface is given by = − 48 .
dt A
Estimate the time taken for the water level to fall from 14 to 10 (units)
above the sluices.

(g) A solid of revolution is formed by rotating about the x-axis, the area
between the x-axis, the lines x = 0 and x = 1 and a curve through the points
with the following co-ordinates.
x 0.00 0.25 0.50 0.75 1.00
y 1.0000 0.9896 0.9589 0.9089 0.8415

Estimate the volume of the solid formed using Simpson’s Rule.


(h) The velocity v (km/min) of a moped at distance x from a point on its path is
given by the following table :
x 0 10 20 30 40
v 45 60 65 54 42

Use Simpson’s rule to find approximately the time taken to traverse the
distance of 40 units.

5.9 SUMMARY
We summarise what we have studied in this unit :
• Discussed the procedure of locating the position of a point in space, and
studied the direction cosines and direction ratios of a line and the properties
associated with them.
• Derived the expression of the angle between the lines and established the
conditions for the perpendicularity and for the parallelism of the lines.
• Derived the equation of the plane in (i) one-point form, (ii) the intercept
form when the intercepts are given on the co-ordinate axis, (iii) normal form
that is the form when the length of the perpendicular from the origin on the
plane as also the direction cosines of this normal are given and (iv) the
general three points form.
122
• Derived formula giving the length of the perpendicular from a point on the Three Dimensional
Geometry
plane.
• Studied the angle between the two planes as also the angle between a line
and a plane and derived expressions for these angles. Discussed the
condition of perpendicularity of two planes and perpendicularity of a line
and a plane.
• Studied surface and solids and related them with polynomial equations
involving one variable, two variables and three variables.
• Derived the equation of a sphere in (i) standard form, (ii) general form and
(iii) in diameter form. Also discussed the procedure of writing the
co-ordinates of the centre of the sphere and its radius when the equation of
the sphere is known.
• Defined the section of a sphere by a plane as also the procedure of finding
the area and the radius of this section.
• Established the condition when a general second degree equation in three
variables x, y, z represents a sphere, i.e. the coefficients of x2, y2 and z2 in the
equation of the sphere and must be equal and the second degree terms xy, yz
and zx must be absent.
• Defined cones and cylinder and also derived the equations of right circular
cone and right circular.
• Defined area bounded by the curve y = f (x), the two ordinates x = a and
x = b and x-axis by a single integral and represented the same area as a
double integral as
b π
⎡ f ( x) ⎤
∫ f ( x) dx = ∫ dx ⎢
⎣ ∫0 dy ⎥

a 0

• Defined volume as a double and volume as a triple integral.


• Studied three systems of co-ordinates namely
(a) Cartesian system (x, y, z)
(b) Cylindrical polar system (ρ, θ, φ), and
(c) Spherical polar system (γ, θ, φ).
and also established the relation between them.
The elements of the volume in the three systems are :
(a) Cartesian system dx dy dz.
(b) Cylindrical polar system ρ dφ dρ dz.
(c) Spherical polar system r dθ dr . r sin θ dφ.
• It is worthwhile remarking here that a certain problems involving multiple
integrals could be simplified and solved by effecting the change of variables
in the integrand from one system of co-ordinates to another system. This has
been shown and discussed through examples.
• Finally the integrals which cannot be solved in closed form are subjected to
numerical method and here in this unit we have studied them by applying
1
Trapezoidal rule, Simpson’s rule.
3
123
Mathematics-I
5.10 ANSWERS TO SAQs
SAQ 1
(b) Coordinates of P (4, 6, 12)
(c) Ratio is 2 : 1
2
(d) −
3
SAQ 2
(b) The equation of the plane is x + 2 y − 2 z + 10 = 0 and the length of the
perpendicular from (1, 4, 5) is 3.
(c) Equation is x + 3 y + z ± 2 11 = 0 .
(d) y + 2z = 4
1 1 1
(e) x + y − z = 1 ; the angles for cos α = ; cos β = ; cos γ = − .
3 3 3
(f) 2x + 3y + 4z = 4
(g) 4 x + 5 y + 3 z = − 11
SAQ 4
(a) (0, − 1, 2); radius = 3.

⎛− 7 − 5 − 2⎞
(b) Centre ⎜ , , ⎟ ; radius = 3.
⎝ 3 2 3 ⎠

Sphere 3 ( x 2 + y 2 + z 2 ) + 14 x + 10 y + 4 z = 1

(c) x2 + y 2 + z 2 ± 6 z − 4 = 0

(d) See the preceding text for proof.


2 4 22 ⎛1 1 2⎞
(e) x2 + y2 + z 2 − x−y− z− = 0 ; Centre ⎜ , , ⎟
3 3 3 ⎝3 2 3⎠

293
Radius =
6
40π
(f) x 2 + y 2 + z 2 + 12 x − 2 y − 6 z + 30 = 0 ; Area of the circle .
3

(g) x2 + y2 + z 2 − 2 x − y = 0

(h) x2 + y 2 + z 2 − 2 x + 4 y − 4z − 4 = 0

SAQ 5

(a) 2 x 2 + 3 y 2 + 5 z 2 − 3 yz − 6 zx + z − 1 = 0

(b) x 2 − 3 y 2 + z 2 − 2 xy + 8 y − 4 = 0

(c) 5 ( x − 1) 2 + 3 ( y − 2) 2 + ( z − 3) 2 − 2 ( y − 2) ( x − 1) − 4 ( z − 3) ( x − 1)

− 6 ( y − 2) ( z − 3) = 0
124
Three Dimensional
(d) 2 x 2 + y 2 + 5 z 2 − 4 xz = 0 Geometry
5
(e) x=± ( z − 1), y = 0
2

(f) x 2 + y 2 + z 2 + 6 ( xy + yz + zx ) − 16 x − 36 y − 4 z = 28

(g) 19 x 2 + 13 y 2 + 3 z 2 − 8 xy − 24 yz − 12 zx = 0

(h) 65 x 2 + 17 y 2 + 17 z 2 − 64 xy − 128 yz − 64 zx
+ 192 x + 222 y + 60 z − 171 = 0
SAQ 6

(a) x 2 + y 2 + xy − x − y = 0
2 2
⎛ 2z ⎞ ⎛ 5z ⎞
(b) 3⎜x − ⎟ + 4 ⎜ y − ⎟ = 12
⎝ 3 ⎠ ⎝ 3⎠

(c) (3 x + z − 1) 2 + (3 y − 2 z + 2) 2 = 8

(d) 2 y 2 + 3 z 2 − 2 yz + 2 y − 2 z − 11 = 0

(e) 3 x 2 + 6 y 2 + 3 z 2 + 8 yz − 2 zx + 6 x − 24 y − 18 z + 24 = 0

(f) 45 x 2 + 40 y 2 + 13 z 2 + 36 yz − 24 zx + 12 xy
− 42 x − 280 y − 126 z + 294 = 0

(g) (2 x + 2 y − 6 z + 3) 2 + 81 = 9 [( x − 1) 2 + ( y − 3) 2 + ( z − 5) 2 ]

(h) x 2 + y 2 + z 2 − 6 x − 6 y − 10 z + 30 = 0
SAQ 7
a2
(a)
2
67
(b)
120
3π a 4
(c)
4
π
(d)
4

π ⎛ a2 a2 ⎞
(e) ⎜⎜ log a − ⎟
2 ⎝ 2 4 ⎟⎠

(f) ab
32
(g)
3
π a3
(h) (i) (2 − 2)
3
2a 3
(ii) (3π − 4)
9
125
Mathematics-I a3
(i) (9π − 16)
36
8
(j) ( a + b) a b
3

3π a 3
(k)
2
4
(l) x = y = 0, z =
3
SAQ 8
(a) (i) 3.04276
(ii) 3.02784
(b) (i) 0.7854, π = 3.14156

(ii) 0.7854, π = 3.14156

(c) 2.8192
(d) 20.15 (cubic ft)
(e) (i) Area under the curve is 11.5
(ii) x = 3.0322 ; y = 1.03565
(iii) 74.7948
(iv) 1.4584
(f) 29 minutes approximately
(g) 2.8192
(h) 0.725 units of time

126

You might also like